Benign and Malignant Neoplasms Flashcards

1
Q

1- An elderly patient with no prior history of skin cancer points out a hyperpigmented skin lesion on the trunk that clinically looks like the most common, non-melanocytic growth found mostly in older individuals. For further evaluation and confirmation, you pull out your dermatoscope as a diagnostic tool. Which of the following features would point away from a seborrheic keratosis?

A- Fissures
B- Fingerprinting
C- Telangiectasias arranged in regular loops
D- Pigment Network
E- Milia-like cysts

A

Correct choice: D. Pigment Network

Explanation: Seborrheic keratosis are the most common benign epidermal tumor which frequently occur in individuals older than 30. Dermoscopic features are comedo-like openings, milia-like cysts, fissures (brain-like cerebriform appearance), fingerprinting, lack of true pigment network, and telangiectasias arranged in regular loops (hair-pin like vessels). The absence of pigment network, branched streaks, and pigment globules are the key features that differentiate SKs from melanocytic neoplasms.

How well did you know this?
1
Not at all
2
3
4
5
Perfectly
2
Q

2- Clinically, this lesion often has a blue hue and usually occurs on the face: A- Apocrine hidrocystoma
B-Epidermoid inclusion cyst
C- Bronchogenic cyst
D- Eccrine hidradenoma
E- Myxoid cyst

A

Correct choice: A. Apocrine hidrocystoma

Explanation: Apocrine hidrocystoma are usually solitary, tranlucent nodules which may have a bluish hue due to the Tyndall effect. Histologically, they have one or several large cystic spaces with decapitation secretion. The other listed tumors are less likely to fit this clinical description.

How well did you know this?
1
Not at all
2
3
4
5
Perfectly
3
Q

3- The most likely diagnosis is?

A- CTCL
B - B-cell lymphoma
C- Angiosarcoma
D- Hemangioma
E- Merkel cell carcinoma

A

Correct choice: B. B-cell lymphoma

Explanation: These red juicy papules are characteristic for B-cell lymphoma. CTCL presents as erythematous scaly patches and plaques. Angiosarcoma is typically a violaceous nodule on the scalp. Merkel cell carcinoma is often a pink-skin colored solitary nodule on the head and neck.

How well did you know this?
1
Not at all
2
3
4
5
Perfectly
4
Q

4- An elderly native Floridian with a fair complexion and evident photodamage due to a history of chronic sun exposure presents for a skin check. He has numerous erythematous, flat, rough macules that are easily felt thorough his head, neck, forearms and dorsal hands. He usually presents to your office every 6 months for treatment of these lesions with cryotherapy. The etiologies responsible for the development of these skin lesions include which of the following?

A- UVA from sunlight
B- Formation of pyrimidine dimers in DNA
C- Mutations in patched within keratinocytes
D- Increasing apoptosis

E- Increased host immune response

A

Correct choice: B. Formation of pyrimidine dimers in DNA

Explanation: The etiology of actinic keratoses involves UVB from sunlight which is responsible for the development of AKs. This triggers the formation of thymidine dimers in DNA & RNA, resulting in mutated keratinocytes. The mutations occur on the tumor suppressor gene p53 within keratinocytes resulting in impaired apoptosis. Clonal expansion of mutated keratinocytes occurs, which may lead to formation of AKs.

How well did you know this?
1
Not at all
2
3
4
5
Perfectly
5
Q

5- All of the following accurately describe properties and mechanism of action of Denileukin diftitox (ONTAK®) EXCEPT:

A- Is a systemic treatment option for CTCL
B- Is a diphtheria fusion toxin
C - Targets the interleukin-2 receptor
D- May cause capillary leak syndrome as an adverse reaction
E- Targets the CD28 protein on T cells

A

Correct choice: E. targets the CD28 protein on T cells

Explanation: Denileukin diftitox is a diphtheria fusion protein chemotherapeutic agent used for the treatment of persistent or recurrent CTCL. It binds selectively to the high- and intermediateaffinity IL-2 receptor (CD25+) on lymphocytes and is internalized by these cells. Inside the cells, the toxin portion of the fusion protein is cleaved by proteolytic enzymes, causing cell death. It produces durable responses and may forestall disease progression. The most frequent and clinically significant adverse events include infusion reactions, capillary leak syndrome, hypoalbuminemia, visual changes, consititutional symptoms, rash, and hepatobiliary disorders - many of these can be managed without dose reduction. Deniluekin diftitox does not target the CD28 expression protein on T cells.

How well did you know this?
1
Not at all
2
3
4
5
Perfectly
6
Q

6- A patient presents for treatment of a melanoma on the cheek with Breslow depth of 0.7 mm and ulceration on pathology. There is no evidence of nodal metastasis. Based on the 8th edition AJCC, what is the correct staging for this patient?

A- Stage 0
B Stage IA
C -Stage IB
D- Stage IIA
E- Stage IIB

A

Correct choice: C. Stage IB

Explanation: Tumors less than 0.8 mm WITH ulceration OR tumors 0.8 - 1mm (regardless of ulceration) are a T stage of T1b. If there is no nodal or distant metstasis then the patient is a cancer StageIB. Patients with tumors 1.01 - 2mm without ulceration are T2a, and if there is no nodal metastasis they are also considered StageIB.
Stage 0 is melanoma in situ (T stage is referred to as Tis).
Stage 1A includes melanomas that are Breslow depth 0.01-0.7 WITHOUT ulceration. Note that tumors should be reported with a single digit after the decimal, and rounded up or down accordingly. Therefore a Breslow of 0.74 mm would be rounded down and reported as 0.7 mm, and a Breslow of 0.75 would be rounded up and reported as 0.8 mm).
Stage IIA would be T2b or T3a and no nodal metastasis. Stage IIB T3b or T4a and no nodal metastasis.

How well did you know this?
1
Not at all
2
3
4
5
Perfectly
7
Q

7- What is true about this tumor?

A- It is the second most common type of skin cancer
B- Telangiectasias are not commonly seen

C- Rombo syndrome has these tumors
D- Seen in Cowden syndrome
E- Mohs micrographic surgery is not an effective treatment

A

Correct choice: C. Rombo syndrome has these tumors

Explanation: This is a classic nodular basal cell. Mohs is often the most effective treatment. Rombo syndrome presents with atrophoderma vermiculatum, basal cell carcinoma, milia, telangiectasias, and acral erythema. The following syndromes can have multiple BCCs: Gorlin, Bazex-Dupre- Christol, Rombo, Brooke-Spiegler, xeroderma pigmentosum, and Schöpf-Schulz-Passarge.

How well did you know this?
1
Not at all
2
3
4
5
Perfectly
8
Q

8- What will the histology of this lesion look like?

A- Small round blue cells with salt and pepper chromatin
B- Basaloid proliferations budding off the epidermis and in the dermis
C- Atypical keratinocytes invading the dermis
D- Lentiginous proliferation of atypical melanocytes
E- Clonal keratinocytes within the epidermis

A

Correct choice: D. Lentiginous proliferation of atypical melanocytes.

Explanation: This is a lentigo maligna melanoma on dermoscopy which will show a lentiginous proliferation of atypical melanocytes along the DEJ and invading into the dermis. It is not a basal cell carcinoma which would be basaloid islands within the dermis, clonal keratinocytes within the epidermis would be a seborrheic keratosis. Merkel cell carcinoma is characterized by salt and pepper chromatin. SCC would be atypical keratinocytes invading the dermis.

How well did you know this?
1
Not at all
2
3
4
5
Perfectly
9
Q

9- Epithelioma cuniculatum is an HPV-associated verrucous carcinoma involving the:

A- Oral mucosa
B- Scalp
C- Sole
D- Penis
E- Trunk

A

Correct choice: C. Sole

Explanation: Verrucous carcinoma is a subtype of SCC that presents with well-defined exophytic growths that can resemble large warts. Lesions are subclassified according to site. Epithelioma cuniculatum is a form of verrucous carcinoma involving the sole. Verrucous carcinoma of the oral mucosa is called oral florid papillomatosis. Verrucous carcinoma of the anogenital area is called giant condyloma acumintum of Buschke-Lowenstein. Verrucous carcinoma of the scalp does not have a unique name.

How well did you know this?
1
Not at all
2
3
4
5
Perfectly
10
Q

10- This is considered to be the juvenile counterpart of DFSP:

A- Juvenile hyaline fibromatosis
B- Giant cell fibroblastoma
C- Myxofibrosarcoma
D- Myofibromatosis
E- Plexiform fibrous histiocytoma

A

Correct choice: B. Giant cell fibroblastoma

Explanation: Giant cell fibroblastoma is CD34-positive, is mostly seen in male children on the neck/trunk, and is thought to be a juvenile counterpart of DFSP. The other listed tumors are not as closely related to DFSP.

How well did you know this?
1
Not at all
2
3
4
5
Perfectly
11
Q

11- This has what finding on dermoscopy?

A- Brown globules

B- Arborizing vessels
C- Blue-grey veil
D- Starburst pattern
E- Red lagoons

A

Correct choice: D. Starburst pattern

Explanation: This is a Spitz nevus. A starburst pattern suggests a Spitz nevus. A basal cell carcinoma which would show arborizing vessels on dermoscopy. A blue-grey veil is associated with melanoma. Brown globules is indicative of pigmented nests in the papillary dermis. Red lagoons indicate a hemangioma.

How well did you know this?
1
Not at all
2
3
4
5
Perfectly
12
Q

12- A 65 year-old woman is referred to your office for evaluation of a skin lesion that has been increasing in size over the past 5 years. You perform a skin biopsy that results as poorly differentiated squamous cell carcinoma (SCC). Risk factors for cutaneous squamous cell carcinoma (cSCC) include all of the following EXCEPT:

A- Kidney disease
B- Fitzpatrick skin types I and II
C- Chronic ulcers
D- Immunosuppression
E- Chronic sun exposure

A

Correct choice: A. Kidney disease

Explanation: Kidney disease in and of itself is not a risk factor for cSCC, however a patient who has had a kidney transplant is at higher risk for the development of cSCC, 65x increased risk over the general population due to the use of immunosuppressive agents to prevent organ rejection. Other risk factors for SCC include: chronic sun exposure, skin types I and II, chemical carcinogens (arsenic, tobacco, coal, tar), immunosuppression, chronic ulcers, burn scars, genetic syndrome (e.g. xeroderma pigmentosa), male gender, and older age. The remaining answer choices are risk factors for SCC.

How well did you know this?
1
Not at all
2
3
4
5
Perfectly
13
Q

13- Dermoscopic features suggestive of malignant melanoma include:

A- Lacunar pattern
B- Arborizing blood vessels
C- Maple leaf structures
D- D Glomeruloid vessels
E- Blue-white veil

A

Correct choice: E. Blue-white veil

Explanation: Asymmetry, multicomponent pattern, blue-whitish veil, parallel-ridge pattern, atypical pigment network, uneven radial streaming, localized irregular and diffuse pigmentation, irregularly distributed globules, and regression structures are all dermoscopic features suggestive of malignant melanoma. Lacunar pattern is seen with cherry angiomas. Arborizing blood vessesl are seen with basal cell carcinomas. Glomeruloid vessels are seen with squamous cell carcinoma in situ. Maple leaf structures are seen with basal cell carcinoma.

How well did you know this?
1
Not at all
2
3
4
5
Perfectly
14
Q

14- Excisional biopsy was performed on this lesion and revealed melanoma in situ. In the photo shown, what is the recommended margin for surgical excision?

A- 0.5-1.0cm
B- 1.0-1.5cm
C- 1.5-2cm
D- 2-2.5cm
E- 3cm

A

Correct choice: A. 0.5-1.0cm

Explanation: Lentigo maligna or melanoma in-situ requires a surgical margin of 0.5-1cm.

How well did you know this?
1
Not at all
2
3
4
5
Perfectly
15
Q

15- Which of the following markers do not stain melanocytic lesions:

A. CK7
B. Vimentin
C. S-100
D. HMB-45
E. All of these answers are correct

A

Correct choice: A. CK7

Explanation: CK7 will stain keratinocytes. Vimentin, S-100 and HMB-45 stains melanocytic lesions.

How well did you know this?
1
Not at all
2
3
4
5
Perfectly
16
Q

16- The treatment for Merkel cell carcinoma is all of the following except:

A. Wide local excision 2-3 cm
B. Mohs
C. Radiation sensitive therapy
D. Chemotherapy
E. Local excision of 1-2 cm

A

Correct choice: A. Wide local excision 2-3 cm

Explanation: Merkel cell carcinoma is a type of aggressive cancer with a high rate of recurrence. It is a neuroendocine carcinoma that is most commonly on sun exposed areas like the head and neck in the elderly. Treatment includes wide local excision of 1-2 cm (per the NCCN guidelines), Mohs, radiation, and chemotherapy.

How well did you know this?
1
Not at all
2
3
4
5
Perfectly
17
Q

17- When performing a biopsy of a suspected keratoacanthoma, which of the following is incorrect?

A- Fusiform incision through the entire KA may be performed
B- It is necessary to biopsy down to subcutaneous fat
C- A full-thickness shave biopsy is acceptable
D- A complete excisional biopsy may be performed
E- It is important to biopsy the lesion, even if it is less than 1cm

A

Correct choice: B. It is necessary to biopsy down to subcutaneous fat

Explanation: When considering a keratoacanthoma, it is not required to obtain a biopsy of the specimen down to the subcutaneous fat. It is important to biopsy all lesions concerning for keratoacanthoma, even those sized less than 1cm. Biopsy can be achieved either by complete excisional biopsy, full-thickness shave biopsy, or fusiform incision through the entire KA including its center and sides.

How well did you know this?
1
Not at all
2
3
4
5
Perfectly
18
Q

19- Your patient with extensive actinic damage and many of the lesions seen here is interested in field therapy with a lightbased modality. On a cellular level, how does photodynamic therapy kill cells that make up this lesion?

A- Heat shock proteins
B- Inhibition of pyrimidine nucleotide synthesis

C- Neutrophil-mediated, antibody-dependent cellular cytotoxicity
D- Reactive singlet oxygen
E- Proteosome inhibition

A

Correct choice: D. Reactive singlet oxygen

Explanation: The lesions seen here is an example of Bowen’s disease (superficial squamous cell carcinoma). Photodynamic therapy is a photochemical reaction in which a photosensitizing molecule, under an activating wavelength, reacts with oxygen in the tissue to create reactive singlet oxygen species. This leads to local necrosis. The other options do not explain how photodynamic therapy works.

How well did you know this?
1
Not at all
2
3
4
5
Perfectly
19
Q

20- What pathway is involved with this tumors growth?

A- Hedgehog pathway
B- RAS pathway
C- WNTpathway
D- ERK pathway
E- MAPK pathway

A

Correct choice: A. Hedgehog pathway

Explanation: Basal cell carcinomas are associated with PTCH mutations which are in the sonic hedgehog pathway.

How well did you know this?
1
Not at all
2
3
4
5
Perfectly
20
Q

21- A patient requests removal of a hyperpigmented plaque with a stuck-on appearance that is eczematous and erythematous in appearance due to trauma. Under the microscope, the hallmark histopathologic findings that you are most likely to see that help distinguish this lesion from other neoplasms include:

A- Papillomatosis, acanthosis, hyperkeratosis
B- Hypogranulosis, hyperkeratosis, pseudoepitheliomatous hyperplasia
C- Hypergranulosis, parakeratosis, acanthosis
D- Parakeratosis, hyperkeratosis, vacuolated keratinocytes
E- Vacuolated keratinocytes, acanthosis, hyperkeratosis

A

Correct choice: A. Papillomatosis, acanthosis, hyperkeratosis

Explanation: Hallmark histopathologic findings of seborrheic keratoses include: acanthosis, papillomatosis, hyperkeratosis, horn cysts and pseudohorn cysts. In contrast to a common wart, vacuolated keratinocytes are not present in stucco keratoses, a clinicopathologic variant of seborrheic keratoses.

How well did you know this?
1
Not at all
2
3
4
5
Perfectly
21
Q

22- Which of the following is true regarding digital HPV associated squamous cell cancers?

A- The rate of metastasis approaches 15%.
B- HPV18 is the most common associated sybtype.
C- Mohs micrographic surgery yields a 20% recurrence rate.
D- Women outnumber men 2:1.
E- These lesions only occur in association with immunosuppression.

A

Correct choice: C. Mohs micrographic surgery yields a 20% recurrence rate.

Explanation: According to Riddel et al (JAAD 2011;64(6):1147- 1153), Mohs micrographic surgery, although the treatment of choice, results in a 20% recurrence rate, which is significantly higher than cutaneous SCC. HPV16 is most often implicated. Men outnumber women 2:1. The rate of metastasis averages between 2-3%. Although common in transplant patients, HPV associated digital SCCs can occur secondary to trauma and in immunocompetent patients.

How well did you know this?
1
Not at all
2
3
4
5
Perfectly
22
Q

23- What is the original function of the gene mutated in this tumor?

A- Inhibit p53
B- Inhibit c-KIT
C- Inhibit smoothened
D- Activate smoothened
E- Inhibit hedgehog

A

Correct choice: C. Inhibit smoothened

Explanation: PTCH is the most common gene mutated in basal cell carcinomas. The normal function of PTCH is to inhibit smoothened. When PTCH is inactivated smoothened is free to activate the sonic hedgehog pathway and stimulate the cell-cycle resulting in tumor cell proliferation.

How well did you know this?
1
Not at all
2
3
4
5
Perfectly
23
Q

24- UVB induced mutations on the PTCH gene is associated with the development of:

A- BCC
B- Merkel cell carcinoma
C- Angiosarcoma
D- BCC and Merkel cell carcinoma
E- Merkel cell carcinoma and Angiosarcoma

A

Correct choice: A. BCC

Explanation: The p53 and PTCH genes are the major targets of UVB for the development of BCC. Other genes involved include the Smoothened-activating mutations, and PTCH2 mutations. The other listed tumors are not due to mutations in the PTCH gene.

How well did you know this?
1
Not at all
2
3
4
5
Perfectly
24
Q

25- Which form of BCC is the most common form in patients with HIV infection?

A- Superficial BCC
B- Nodular BCC
C- Morpheaform BCC
D- Infiltrative BCC
E- Micronodular BCC

A

Correct choice: A. Superficial BCC

Explanation: Superficial BCC is the most common form of BCC seen in patients with HIV.

How well did you know this?
1
Not at all
2
3
4
5
Perfectly
25
Q

26- A 56-year-old marathon runner presents with a superficial BCC On the left lower leg. He prefers to avoid excision or dessication and curettage and presents 4 weeks later with the findings seen here. Which of the following is the best statement regarding counseling for the patient?

A- Treatment with topical creams for superficial cancers has a lower cure rate than surgery but has an excellent cosmetic outcome. The findings here are consistent with a robust response and suggests a better clearance of the tumor at completion of therapy

B- The tumor here has grown significantly and the patient must now undergo Mohs surgery for therapy given the size and location of the lesion

C- Desiccation and curettage has a similar cosmetic outcome to topical therapies (5FU or imiquimod) as well as similar cure rate to excision

D- Mohs surgery is never appropriate for superficial basal cell or squamous cell carcinoma in situ malignancies as they are minimally invasive

E- It is best to monitor the superficial basal cell cancer annually because it is not an aggressive malignancy

A

Correct choice: A. Treatment with topical creams for superficial cancers has a lower cure rate than surgery but has an excellent cosmetic outcome. The findings here are consistent with a robust response and suggests a better clearance of the tumor at completion of therapy.

Explanation: Imiquimod can be a reasonable treatment for superficial basal cell cancers. It has an excellent cosmetic outcome but can be very inflammatory, as seen in the image. The tumor has not grown significantly, this is a reaction to the imiquimod. Dessication and curettage often has a worse cosmetic outcome than topical chemotherapeutics and has a lower cure rate than excision. Based on location and size, Mohs surgery is sometimes an appropriate option for superficial keratinocyte cancers. It is not unreasonable to monitor a superficial BCC as they are slow growing tumors, however it is not the best answer and annual follow-up may not be enough treatment.

How well did you know this?
1
Not at all
2
3
4
5
Perfectly
26
Q

27- Where is the classic location of this lesion?

A- Scalp
B- Chest
C- Back
D- Nose
E- Ear

A

Correct choice: A. Scalp

Explanation: This is an angiosarcoma as evident by the numerous atypical vessels, hemorrhage and atypical endothelial cells within the atypical vessels. These tumors most commonly occur on the scalp as violaceous nodules in elderly men.

How well did you know this?
1
Not at all
2
3
4
5
Perfectly
27
Q

28- The application of what topical medication has been shown to decrease the risk of developing squamous cell carcinomas in the first year after treatment?

A- Ingenol mebutate
B- Imiquimod
C- Tacrolimus
D- Diclofenac
E- Fluorouracil 5%

A

Correct choice: E. fluorouracil 5%

Explanation: Fluorouracil 5%. A standard course of fluorouracil treatment (application twice daily for 2 to 4 weeks) to the face and ears were shown to decrease the risk of SCC by 75% in the first year after treatment, although it was not shown to decrease the risk of keratinocyte carcinomas over a 4-year period. The remaining medications are FDA-approved for the treatment of actinic keratosis, but have not yet been studied for chemoprevention of basal or squamous cell carcinomas.

How well did you know this?
1
Not at all
2
3
4
5
Perfectly
28
Q

29- Where is the classic location of this lesion?

A- Forehead
B- Dorsal nose

C- Upper cutaneous lip at the base of the nasal ala
D- Ear
E- Chin

A

Correct choice: C. Upper cutaneous lip at the base of the nasal ala

Explanation: This is a microcystic adnexal carcinoma. These typically occur on the upper cutaneous lip at the base of the nasal ala in young adult females. It is imperative the base of the tumor be seen to ensure it is completely removed as the tumor is aggressive.

How well did you know this?
1
Not at all
2
3
4
5
Perfectly
29
Q

30- What will this show on dermoscopy?

A- Arborizing vessels
B- Red lagoons
C- Blue-grey veil
D- Brown globules
E- Starburst pattern

A

Correct choice: A. Arborizing vessels

Explanation: This is a basal cell carcinoma which will show arborizing vessels. A hemangioma will show red lagoons on dermoscopy. A spitz nevus has a starburst pattern. A blue-grey veil is associated with melanoma. Brown globules is indicative of pigmented nests in the papillary dermis.

How well did you know this?
1
Not at all
2
3
4
5
Perfectly
30
Q

31- All of the following statements are true regarding the ‘MCW Melanoma Cocktail’ except:

A- it is a mixture of monoclonal antibodies that include MART-1
B- it is performed intraoperatively
C- it is a mixture of monoclonal antibodies that include Melan-A
D- it demonstrates micrometastases in sentinel lymph nodes
E- it is a mixture of polyclonal antibodies to tyrosinase

A

Correct choice: E. it is a mixture of polyclonal antibodies to tyrosinase

Explanation: The ‘MCW Melanoma Cocktail’ is an immunostain made up of monoclonal antibodies to MART-1, Melan-A and tryrosine. It is used intraoperatively during sentinel lymph node biopsy and allows for rapid and accurate determination of micrometastases. The mixture of stains are monoclonal antibodies; not polyclonal.

How well did you know this?
1
Not at all
2
3
4
5
Perfectly
31
Q

32- What is the mechanism of action of vorinostat, a therapy for refractory cutaneous T cell lymphoma?

A- Histone deacetylase inhibitor
B- IL-2 receptor inhibitor
C- Antibody against CTLA-4
D- JAK 1/3 inhibitor
E- Alkylating agent

A

Correct choice: A. Histone deacetylase inhibitor

Explanation: Vorinostat and romidepsin are histone deacetylase (HDAC) inhibitors. The most common side effects are fatigue, gastrointestinal symptoms and reversible thrombocytopenia. Denileukin difitox is an IL-2 receptor inhibitor, and is associated with capillary leak syndrome. Ipilimumab is an antibody to CTLA-4. Tofacitinib is a JAK 1/3 inhibitor.

How well did you know this?
1
Not at all
2
3
4
5
Perfectly
32
Q

33- Spindle cell lipoma is most commonly found on the:

A- Head
B- Lower extremities
C- Buttocks
D- Breast
E- Posterior shoulder

A

Correct choice: E. Posterior shoulder

Explanation: Spindle cell lipoma is a solitary benign tumor seen in adult male patients, and is most often located on the posterior shoulder or neck regions. The tumor histologically consists of mature collagen, adipose tissue, spindle cells, and mast cells. Treatment is with local excision. None of the remaining answer choices represent the most common location for spindle cell lipoma.

How well did you know this?
1
Not at all
2
3
4
5
Perfectly
33
Q

34- This tumor commonly results from mutations in p53. Which of the following is correct?

A- It is the most common type of skin cancer in Caucasians
B- Oral variants most commonly present on the dorsal tongue
C- It is typically less aggressive in patients with CLL
D- Organ transplant patients have a 10-fold increased risk of developing this tumor
E- Tumors arising within scars/chronic ulcers have the highest risk of metastasis

A

Correct choice: E. Tumors arising within scars/chronic ulcers have the highest risk of metastasis Explanation: This is SCC of the ear. SCCs arising within scars/chronic ulcers metastasize in
~30-40% of cases, whereas those arising on the lip or ear have a 10-20% risk of metastasis. Recurrent SCCs have a metastasis risk of up to 30%. SCC is the 2nd most common skin cancer in Caucasians (BCC is most common). Oral SCC most commonly presents on the lateral tongue. SCC

is typically more aggressive in patient with CLL. Organ transplant patients have a 65-fold increased risk for developing cutaneous SCC.

How well did you know this?
1
Not at all
2
3
4
5
Perfectly
34
Q

35- Which of the following is true about this neoplasm?

A- A sentinel lymph node biopsy is standard of care for tumors thicker than 1 mm
B- Locally advanced or metastatic variants may respond to sonic hedgehog inhibitors
C- Grows very rapidly over several weeks/months before stabilizing
D- Associated with a polyomavirus infection
E- Occurs at sites of chronic burns or fistulae

A

Correct choice: B. Locally advanced or metastatic variants may respond to sonic hedgehog inhibitors

Explanation: The dermoscopic picture is of a pigmented BCC.
Answer choice 1 refers to melanomas; choice 3 refers to keratoacanthoma; choice 4 refers to merkel cell carcinoma; choice 5 refers to squamous cell carcinoma.

How well did you know this?
1
Not at all
2
3
4
5
Perfectly
35
Q

36- The green color in chloroma is secondary to:

A- Stromelysin
B- Chloracetate
C- Fumarase
D- Myeloperoxidase
E- Alkaline phosphatase

A

Correct choice: D. Myeloperoxidase

Explanation: Chloromas are greenish tumor grossly secondary to involvement of the skin in acute granulocytic leukemia. The green color is secondary to myeloperoxidase. The other listed answers do not contribute to the green color of chloroma.

How well did you know this?
1
Not at all
2
3
4
5
Perfectly
36
Q

37- Which of the following is FALSE?

A- These lesions are often mistaken for basal cell carcinomas.
B- In the absence of ulceration, sentinel lymph node biopsy should be considered if tumor thickness is 0.8mm or greater.
C- These lesions are not treated differently than their pigmented variants.

D- These lesions often have a better prognosis than their pigmented variants.
E- Increased tumor thickness is associated with a worse prognosis.

A

Correct choice: D. These lesions often have a better prognosis than their pigmented variants.

Explanation: The correct answer is D (D is false). Amelanotic melanomas are often diagnosed later (i.e. thicker Breshlow depth) than their pigmented counterparts, which is the reason for their poorer prognosis. Amelanotic melanomas are often mistaken for basal cell carcinomas as both can appear clinically similar. The recently published AJCC 8th Edition Melanoma Staging System revised the definitions of T1a and T1b so that T1a melanomas include those <0.8 mm without ulceration while T1b melanomas include those 0.8-1 mm with or without ulceration and those <0.8 mm with ulceration. Mitotic rate is no longer a T1 category criterion but should be documented for all invasive primary melanomas. Thus, in the absence of ulceration, sentinel lymph node biopsy should be considered if tumor thickness is 0.8mm or greater. Amelanotic melanomas are treated identically

to their pigmented counterparts, and increased tumor thickness is associated with a worse prognosis (irrespective of the clinical subtype of melanoma).

How well did you know this?
1
Not at all
2
3
4
5
Perfectly
37
Q

38- This lesion may be associated with which of the following?

A. Cowden disease
B. Rombo syndrome
C. Reed’s syndrome
D. Brooke-Spiegler syndrome
E. Gorlin syndrome

A

Correct choice: D. Brooke-Spiegler syndrome

Explanation: This is a biopsy of a cylindroma. Multiple cylindromas seen with CYLD mutation either in cylindromatosis or in conjunction with other adnexal neoplasms in Brooke-Spiegler syndrome.

How well did you know this?
1
Not at all
2
3
4
5
Perfectly
38
Q

39- A 40 year-old man presents with a slow-growing, large, firm, nodular tumor on the back. Immunohistochemical staining of the biopsy specimen from this tumor is positive for CD34 and negative for Factor XIIIa. The patient subsequently undergoes wide excision with adjuvant chemotherapy. Which of the following chemotherapeutic agents was most likely used?

A. Imatinib
B. Vismodegib
C. Avelumab
D. Ipilimumab

E. Nivolumab

A

Correct choice: A. Imatinib

Explanation: Imatinib has been used in the treatment of primary or locally recurrent dermatofibrosarcoma protuberans. Even with wide excision, the 5 year recurrence rate for this tumor can be 20-25%. This neoplasm commonly possesses a reciprocal translocation t(17;22) resulting in fusion of collagen 1alpha1 and platelet derived growth factor B (a fusion oncogene). Activation of the platelet derived growth factor receptor associated with overexpression of platelet derived growth factor is central to the development of DFSP. Imatinib is a protein tyrosine kinase inhibitor used primarily to treat chronic myelogenous leukemia with the Philadelphia chromosome defect. Imatinib also inhibits the tyrosine kinases associated with platelet derived growth factor and stem cell factor. Therefore, it directly inhibits the platelet derived growth factor receptor signaling cascade, which plays a critical role in the pathogenesis and growth of DFSPs. While Imatinib has been successful in clinical trials, it is not yet FDA-approved for the treatment of DFSP. DFSP stains positive for CD34 and negative for Factor XIIIa, which differentiates it from a dermatofibroma (stains positive for Factor XIIIa and negative for CD34).

Vismodegib is a smoothened-inhibitor that is FDA-approved for the treatment of basal cell carcinoma. Avelumab is an anti-PD-L1 monoclonal antibody that was recently FDA-approved for the treatment of merkel cell carcinoma. Ipilimumab is an anti-CTLA-4 monoclonal antibody that is FDA-approved for the treatment of melanoma. Nivolumab is an anti-PD-1 monoclonal antibody that is FDA-approved for the treatment of melanoma.

How well did you know this?
1
Not at all
2
3
4
5
Perfectly
39
Q

40- Typical dermoscopic features of this common benign lesion that typically begins to appear during the fourth decade of life include all of the following except:

A. Fat fingers
B. Reticulated pigment network

C. Milia-like cysts
D. Cerebriform surface
E. Light brown parallel structures

A

Correct choice: B. Reticulated pigment network

Explanation: Typical dermoscopic features of seborrheic keratoses include: milia-like cysts, irregular crypts, fissures/ridges, blue-gray lobules, light brown fingerprint-like parallel structures, fat fingers (the gyri of a cerebriform surface). A reticulated pigment network is not a common feature of seborrheic keratoses.

How well did you know this?
1
Not at all
2
3
4
5
Perfectly
40
Q

41- Which of the following would you expect to find on dermoscopy of this lesion?

A. Blue-grey ovoid nests

B. Milky red globules
C. Orange crust
D. Maple leaf structures
E. Spoke wheel vasculature

A

Correct choice: B. Milky red globules

Explanation: This is an image of melanoma, which is associated with milky red globules on dermoscopy.

How well did you know this?
1
Not at all
2
3
4
5
Perfectly
41
Q

42- Which of the following is TRUE regarding this lesion?

A. Ulceration is the most important prognostic factor
B. Immunosuppression is not a risk factor
C. Female gender is a poor prognostic factor
D. It is the most common type seen in darker-skinned patients
E. Anti-PD-1 antibodies are not helpful in treatment

A

Correct choice: D. It is the most common type seen in darker-skinned patients

Explanation: The associated image depicts an acral lentiginous melanoma (ALM), which is the most common type of melanoma in darker-skinned individuals. Notably, and very sadly, the famous musician Bob Marley was diagnosed with ALM in 1977 and succumbed to this malignancy in 1981 as it had metastasized to his lungs and brain.

Breslow depth, not ulceration, is the most important prognostic factor for melanoma. Immunosuppression increases one’s risk of melanoma by a factor of 3-4. Male gender is a poor prognostic factor. Anti-PD-1 antibodies (e.g. nivolumab, pembrolizumab) are proven to prolong survival in patients with metastatic melanoma.

How well did you know this?
1
Not at all
2
3
4
5
Perfectly
42
Q

43- This lesion is most likely to possess a mutation in which of the following?

A. NRAS
B. BAP-1
C. CDKN2A
D. BRAF
E. C-kit

A

Correct choice: D. BRAF

Explanation: The pictured lesion is a melanoma. The most common mutation in a melanoma is in the BRAF gene (~50% of advanced or unresectable melanomas). NRAS mutations are the next most commonly found mutation (~30% of melanomas). BAP-1 (BAP-1 tumor predisposition syndrome), CDKN2A (melanoma pancreatic cancer syndrome), and C-kit may also be mutated in cases of melanoma.

How well did you know this?
1
Not at all
2
3
4
5
Perfectly
43
Q

44- This syndrome presents with multiple keratoacanthomas appearing suddenly during childhood or adolescence and inherited in an autosomal dominant pattern:

A. Ferguson-Smith
B. Grzybowski
C. Gorlin
D. KA centrifugum
E. Buschke-Lowenstein

A

Correct choice: A. Ferguson-Smith

Explanation: Patients that have a sudden appearance during childhood of multiple keratoacanthomas have Ferguson-Smith type of KA. This is autosomal dominant and are the KAs are typically self-healing.

Grzybowski is typically diagnosed in adulthood with the sudden appearance of hundreds to thousands of lesions in a disseminated fashion.
Gorlin syndrome is characterized by the appearance of multiple BCCs (not KAs) during childhood. It is also characterized by odontogenic keratocysts of the jaw and skeletal defects (e.g., macrocephaly, hypertelorism, frontoparietal bossing, spina bifida, or rib abnormality, among others). Tumors associated with this disease include medulloblastoma, meningioma, ovarian fibromas (bilateral), and cardiac fibromas. It is inherited in an autosomal dominant pattern, and due to a mutation in PTCH gene.

KA centrifugum is a subtype of solitary KA that may reach a size of up to 20 cm in diameter. Buschke-Lowenstein tumor is a verrucous carcinoma caused by HPV 6 and 11 that is locally invasive and destructive but rarely metastatic.

How well did you know this?
1
Not at all
2
3
4
5
Perfectly
44
Q

45- Biopsy of one of the lesions pictured reveals spindle cells forming slit-like vascular channels. Which of the following is also associated with the cause of these lesions?

A. Bacillary angiomatosis
B. Merkel cell carcinoma
C. Angiosarcoma
D. Multicentric Castleman disease
E. Nasopharyngeal carcinoma

A

Correct choice: D. Multicentric Castleman disease

Explanation: The clinico-pathologic correlation is best for Kaposi sarcoma, which is caused by HHV-8 (KSHV). Multicentric Castleman disease and primary effusion lymphoma are also associated with HHV-8. The other remaining answer choices are not associated with HHV-8.

How well did you know this?
1
Not at all
2
3
4
5
Perfectly
45
Q

46- What is the likely etiology of the pictured lesion?

A. PTCH mutation
B. CDKN2A mutation
C. Lichenoid infiltrate
D. HPV 5
E. p53 mutation

A

Correct choice: E. p53 mutation

Explanation: This is a SCC of the tongue. SCCs most commonly have mutations in p53.
PTCH mutations are in basal cell carcinomas. CDKN2A mutations are in familial melanomas. A lichenoid infiltrate would be seen with lichen planus which is usually on the buccal mucosa. Heck’s disease is due to HPV 13 and 32.

How well did you know this?
1
Not at all
2
3
4
5
Perfectly
46
Q

47- You perform a biopsy of a pink, soft, dome-shaped papule and the histology reveals a predominantly intradermal melanocytic proliferation composed of nests and sheets of epithelioid melanocytes with large pleomorphic nuclei and light eosinophilic cytoplasm. Immunohistochemical staining is positive for BRCA1 associated protein-1. Which of the following malignancies is this patient most at risk of developing?

A. Breast cancer
B. Ovarian cancer
C. Hodgkin’s lymphoma
D. Mesothelioma
E. Pancreatic cancer

A

Correct choice: D. Mesothelioma

Explanation: This clinico-pathologic picture is most consistent with the BRCA1 associated protein-1 (BAP1) cancer syndrome. BAP1 is a nuclear deubiquitinating enzyme that functions as a tumor suppressor via its role in DNA damage repair. The BAP1 cancer syndrome is a rare autosomal dominant genetic syndrome typified by the development of mesotheliomas and uveal melanomas. Patients with BAP1 cancer syndrome develop several, distinct, melanocytic neoplasms. These are raised, pink or tan, dome-shaped, benign lesions that have been referred to in the literature as “melanocytic BAP1-mutated atypical intradermal tumors/MBAITs” or “BAPomas.”

Less commonly, cutaneous melanomas, various types of carcinomas (mostly from the kidney and gallbladder), sarcomas, and brain tumors can arise in the BAP1 cancer syndrome. Thus, this patient is less likely to develop one of the other listed answer choices compared to mesothelioma.

How well did you know this?
1
Not at all
2
3
4
5
Perfectly
47
Q

48- The clear-cell variant of this neoplasm is associated with which of the following?

A. Brooke-Spiegler syndrome
B. Cowden disease
C. Rombo syndrome
D. Diabetes mellitus
E. Hypothyroidism

A

Correct choice: D. Diabetes mellitus

Explanation: This is a syringoma. The clear-cell variant (a rare histological variant of syringoma that is clinically indistinguishable from an ordinary syringoma) has been associated with diabetes mellitus.

How well did you know this?
1
Not at all
2
3
4
5
Perfectly
48
Q

49- What is the most common gene mutated in this type of lesion?

A. PMS
B. c-KIT
C. p53
D. MSH
E. PTCH

A

Correct choice: E. PTCH

Explanation: This is a basal cell carcinoma. The most common gene mutation is PTCH.
p53 is mutated in SCCs, PMS2 and MSH6 are lost in sebaceous neoplasms of Muir-Torre. c-KIT is mutated in acral melanoma, mucosal melanoma, and mast cell disorders.

How well did you know this?
1
Not at all
2
3
4
5
Perfectly
49
Q

50- Which of the following treatments carries the lowest risk of recurrence in the condition pictured?

A. Pembrolizumab
B. Imatinib mesylate
C. Radiation therapy
D. Mohs micrographic surgery
E. Wide local excision

A

Correct choice: D. Mohs micrographic surgery

Explanation: Standard treatment of dermatofibrosarcoma protuberans (DFSP) consists of complete surgical extirpation. The rate of recurrence is lower following Mohs micrographic surgery compared to wide local excision.

This question asks the examinee to identify DFSP presenting as a characteristic firm red-brown tumor on the shoulder, and know the treatment that will impart lowest recurrence risk. Initial treatment of this tumor is complete surgical removal, with studies demonstrating lower risk of recurrence with Mohs micrographic surgery (choice 4) versus wide local excision (choice 5). The programmed cell death-1 inhibitor pembrolizumab (choice 1) has not been used to treat DFSP. Imatinib mesylate (choice 2), which interferes with the activity of the COL1A1-PDGFB fusion protein found in DFSP, is FDA-approved for unresectable, recurrent, and metastatic cases. Radiation (choice 3) has been utilized for unresectable or recurrent DFSP, and would not be an appropriate choice for initial therapy.

How well did you know this?
1
Not at all
2
3
4
5
Perfectly
50
Q

51- Merkel cell carcinoma should be treated with what size surgical margins?

A. 2mm
B. 5mm
C. 1-2cm to subcutaneous
D. 1-2cm to fascia
E. 5cm

A

Correct choice: D. 1-2cm to fascia

Explanation: Merkel cell carcinoma is an aggressive rare tumor of the skin accounting for less than 1% of cutaneous malignancies. Also known as neuroendocrine cancer of the skin, this tumor presents as a painless red to violaceous, firm, solitary, nodule that usually presents on sun exposed areas such as the head, neck and upper extremities. These tumors present usually during the 6th and 7th decades, and have a 2 year survival rate of 50-70%. Because of this tumor’s high potential for regional and distal metastasis, this tumor should be excised with wide local excision with 1-2cm surgical margins to the depth of fascia or pericranium, or treated with Mohs Surgery.

How well did you know this?
1
Not at all
2
3
4
5
Perfectly
51
Q

52- On dermoscopy, which of the following findings most strongly supports the correct diagnosis?

A. Starburst pattern
B. Blue-whitish veil
C. Maple leaf structures
D. Regression structures
E. Irregular streaks

A

Correct choice: C. Maple leaf structures

Explanation: Maple leaf structures on dermoscopy support the diagnosis of basal cell carcinoma.

This question asks the examinee to identify a pigmented BCC and select the most closely associated dermoscopic finding. In the case of this dark blue-black irregular papule at the alar crease, initial concern for melanoma is quite reasonable. However, on close inspection, there is a subtly rolled border, suggesting the correct diagnosis, pigmented BCC. Dermoscopy can assist in distinguishing between pigmented BCCs and other pigmented lesions. Among the answer choices, maple leaf structures (choice 3) are most closely associated with BCC. The starburst pattern (choice 1) is classically seen in pigmented spindle cell nevus of Reed. Blue-whitish veil (choice 2), regression structures (choice 4), and irregular streaks (choice 5) are features of melanoma, not routinely observed in pigmented BCC.

How well did you know this?
1
Not at all
2
3
4
5
Perfectly
52
Q

53- The keratoacanthoma variant characterized by the sudden appearance during childhood or adolescence of multiple KAs is called:

A. Gorlin syndrome
B. Xeroderma pigmentosa
C. Ferguson-Smith
D. Grzybowski
E. None of these answers are correct

A

►C

The Ferguson-Smith is a keratoacanthoma variant characterized by the sudden appearance during childhood or adolescence of multiple KAs that may resolve and later-on reappear. This condition is inherited in an autosomal dominant pattern

How well did you know this?
1
Not at all
2
3
4
5
Perfectly
53
Q

54- The treatment of choice for this lesion shown is:

A. Radiation therapy
B. Imiquimod
C. 5 Flourouracil
D. Wide excision with 2cm margins
E. Mohs surgery

A

►E

The treatment of choice for Dermaotfibrosarcoma protuberans is Mohs surgery. Radiation therapy has been used, however has limited value as solitary therapy for thsi tumor. Radiation therapy can be used as an adjunct to wide surgical excision. Classically, these tumors should be excised with 3cm margins. The recurrence rate associated with these tumors can be 10 -20 percent with wide excision with 3 cm margins. With Mohs surgery, the recurrence rate ranges from 0% to 6%. 5FU and Imiquimod are not effective modalitites in treating DFSPs, as it infiltrates deep into the subcutaneous tissue.

How well did you know this?
1
Not at all
2
3
4
5
Perfectly
54
Q

55- Mutations in which gene would likely be found in the neoplastic cells of this lesion?
A. PATCH

B. p53
C. Fumarate hydratase
D. CREBBP
E. p63

A

►B

Squamous cell carcinoma is the second most common cancer of the skin. Mutations in the tumor suppressor p16 and p53 are commonly found in SCC’s. Normally, UV damage upregulates p53 thereby delaying cell cycle progression. DNA damage can then be repaired or the cell could undergo apoptosis. In squamous cell carcinoma, p53 exhibits loss of heterozygosity due to C to T or CC to TT mutations.

How well did you know this?
1
Not at all
2
3
4
5
Perfectly
55
Q

56 -A patient with a innumerable disseminated keratoacanthomas, including lesions on the larynx and oral mucosa:

A. Is unlikely to have palmoplantar involvement
B. Likely has an underlying immune deficiency
C. Is at high risk for myelodysplasia
D. Likely inherited their condition in an autosomal dominant manner
E. Likely developed them during adulthood

A

►E

This patient has the Grzybowski type of keratoacanthomas. Typically diagnosed in adulthood, these patients have the sudden appearance of hundreds of small lesions in a disseminated

fashion. The lesions can be found anywhere on the body including palms, soles, larynx, and oral mucosa.

How well did you know this?
1
Not at all
2
3
4
5
Perfectly
56
Q

57 -The patient is a 45 year old male complaining of red, chapped lower lip. Which of the following lasers is the most appropriate to treat this condition?

A. Pulsed Dye Laser
B. Nd:YAG laser
C. CO2 laser
D. Diode laser
E. Laser treatment is not an option

A

►C

The patient has actinic cheilitis. Notice the red, scaly lower lip, with erosions and fissures. The CO2 laser is currently a common treatment alternative for this condition.

How well did you know this?
1
Not at all
2
3
4
5
Perfectly
57
Q

Which of the following ethnic groups are commonly diagnosed with dermatosis papulosa nigra:

A. Asians
B. Hispanics
C. African-Americans & Hispanic patients
D. Caucasians
E. No difference between ethnic groups

A

►C

Characterized by the presence of multiple, small, hyperpigmented, sessile SKs on the face, DPN is typically diagnosed on the African-American and Hispanic population.

How well did you know this?
1
Not at all
2
3
4
5
Perfectly
58
Q

59- Intermittent sun exposure with painful sunburns is a predisposing factor for the development of:

A. Atypical nevi
B. Seborrheic keratosis
C. Malignant melanoma
D. Atypical nevi and Malignant melanoma
E. All of these answers are correct

A

►D

It has been reported that the risk for the development of atypical nevi and melanoma is higher than twofold with a history of five or more episodes of painful sunburn during adolescence.

How well did you know this?
1
Not at all
2
3
4
5
Perfectly
59
Q

60- The following lesion is the classic presentation of:

A. BCC
B. Merkel cell carcinoma
C. Melanoma
D. CTCL
E. Keratoacanthoma

A

►E

Keratoacanthomas present as a solitary, firm, dome-shaped papule with a cratiform center.

How well did you know this?
1
Not at all
2
3
4
5
Perfectly
60
Q

61 -A patient having Mohs surgery for a squamous cell carcinoma on the ear has tumor invading the cartilage and perineural invasion. What stage disease does the patient have?

A. T0
B. T1
C. T2
D. T3
E. T4

A

►C

TX Primary tumor cannot be assessed T0 No evidence of primary tumor Tis Carcinoma in s itu T1 Tumor ≤2 cm in greatest dimension with <2 high-risk features∗ T2 Tumor >2 cm in greatest dimension with or without one additional high-risk feature, or tumor any size with ≥2 high-risk features∗ T3 Tumor with invasion of maxilla, mandible, orbit, or temporal bone T4 Tumor with invasion of skeleton (axial or appendicular) or perineural invasion of skull base. *High risk
features: Depth/invasion >2 mm thickness or Clark level ≥IV,Perineural invasion, Primary site ear, Primary site hair-bearing lip, Poorly differentiated or undifferentiated.

How well did you know this?
1
Not at all
2
3
4
5
Perfectly
61
Q

62- What would you expect to see under dermoscopy of this vascular neoplasm?

A. Red sacculae
B. Arborizing blood vessels
C. Hair pin telangectasia
D. Milky red globules
E. Blue-grey ovoid nests

A

►A

Hemangioma have a characteristic appearance under dermoscopy. Typically, they have a maroon lagoon or red sacculae appearance.

How well did you know this?
1
Not at all
2
3
4
5
Perfectly
62
Q

63- The most important mutated gene associated with a predisposition to develop malignant melanoma is:

A. PTCH
B. CDKN2A
C. PTCH2
D. None of these answers are correct
E. All of these answers are correct

A

►B

The CDKN2A gene located on chromosome 9p21 is the most important mutated gene associated with MM of the listed choices. BRAF is another important mutation.

How well did you know this?
1
Not at all
2
3
4
5
Perfectly
63
Q

64 -A patient is diagnosed with squamous cell carcinoma. As a doctor you explain to the patient that the metastatic rate is:

A. 0.3-16%
B. 15%-20%
C. 18%-25%
D. 25%-30%
E. 30%-45%

A

►A

Patients that have SCC have a 0.3-16% of chance of metastasis. The location, size and type of SCC can determine the possibility of metastasis.

How well did you know this?
1
Not at all
2
3
4
5
Perfectly
64
Q
  1. Which of the following is the most common initial site of metastasis from a primary BCC?

A. Lungs
B. Regional lymph nodes
C. Bone
D. Liver
E. Pleura

A

►B

The metastatic potential of BCC is very low with rates ranging from 0.0028 to 0.1%. The head and neck region is the most frequent location of the primary tumor with regional lymph nodes being the most common site of metastasis. The lungs, bone, liver, and pleural are also potential sites of metastasis.

How well did you know this?
1
Not at all
2
3
4
5
Perfectly
65
Q

66 -Chloroma is a characteristic cutaneous manifestation of:

A. Tuberous sclerosis
B. Sweetǁs syndrome
C. Neurofibromatosis
D. Leukemia
E. Pseudomonas sepsis

A

►D

Chloromas, also termed granulocytic sarcomas, are a localized tumor composed of immature granulocytic cells. They frequently have a greenish coloration due to the presence of myeloperoxidase and most commonly affect the bone. The condition most often occurs in patients with acute leukemia of the myeloid type.

How well did you know this?
1
Not at all
2
3
4
5
Perfectly
66
Q

67- Which phase of the cell cycle does p53 regulate?

A. G1
B. G2
C. S phase
D. Mitosis

E. Meiosis

A

►A

p53 is a tumor suppressor gene which arrests cell cycle in G1 as it controls the transition from G1 to
S. It also downregulates BCL-2. Mutations in p53 are associated with Li-Fraumeni syndrome as well as the development of squamous cell carcinomas.

How well did you know this?
1
Not at all
2
3
4
5
Perfectly
67
Q

68- A 65 year-old female with multiple actinic keratosis on the face under treatment with 5-FU. According to the image and aforementioned information, you may conclude that:

A. The patient must immediately stop treatment since unexpected side effects have developed
B. The patient has been compliant with 5-FU treatment and the appearance of inflammation, erythema and erosions are expected
C. The image is not relevant to 5-FU treatment
D. None of these answers are correct
E. All of these answers are correct

A

►B

Compliance is a key feature in treatment with 5-FU. Erythema, inflammation and erosion must develop and is considered a sign of successful treatment.

How well did you know this?
1
Not at all
2
3
4
5
Perfectly
68
Q

69- Which of the following would you not expect to see under dermoscopy?

A. Maple leaf pattern
B. Arborizing blood vessels
C. Blue-grey ovoid nests
D. Orange crust
E. Milky red globules

A

►E

Dermoscopy is a useful tool in differentiating a pigmented basal cell carcinoma from melanoma. Basal cell carcinomas may have arborizing blood vessels, maple leaf pattern, blue-grey ovoid nests, and orange crust or ulcer. Milky red globules are sometimes seen in melanoma.

How well did you know this?
1
Not at all
2
3
4
5
Perfectly
69
Q

70- Which of the following melanoma subtypes is more consistently seen in dark-skinned individuals?

A. Acral lentiginous melanoma
B. Superficial spreading melanoma
C. Lentigo maligna melanoma
D. Nodular melanoma
E. Superficial spreading melanoma and Lentigo maligna melanoma

A

►A

Acral lentiginous melanoma is the predominant type of melanoma in dark-skinned individuals. It is usually located on the soles, palms, and subungeal region of patients in their fifth to sixth decade of life.

How well did you know this?
1
Not at all
2
3
4
5
Perfectly
70
Q

71- Which of the following is an immunhistochemical marker for Merkel Cell Carcinoma?:

A. S-100
B. Vimentin
C. HMB-45
D. Neuron specific enolase
E. All of these answers are correct

A

►D
Neuron specific enolase stains merkel cells. Vimentin stains melanocytic lesions, sarcomas and lymphomas. S-100 and HMB-45 stains melanocytic lesions, such as melanoma.

How well did you know this?
1
Not at all
2
3
4
5
Perfectly
71
Q

72- Which one of the following malignancies is associated with HPV infection?

A. Verrucous carcinoma
B. Metastatic melanoma
C. Basal cell carcinoma
D. Sebaceous carcinoma
E. Atypical fibroxanthoma

A

►A

Verrucous carcinomas are low-grade carcinomas which are slow-growing and metastasize very late in the course. The presence of HPV has been demonstrated in cases both by electron microscopy and DNA hybridization.

How well did you know this?
1
Not at all
2
3
4
5
Perfectly
72
Q

73- Dermatofibrosarcoma protuberans is:

A. Cytogenetically characterized by reciprocal translocation t(17;22)(q22;q13)
B. Factor XIIIa positive
C. CEA positive
D. CD 34 positive
E. Cytogenetically characterized by reciprocal translocation t(17;22)(q22;q13) and CD 34 positive

A

►E

A chromosomal reciprocal translocation t(17;22)(q22;q13), and supernumerary ring chromosome have been reported as cytogenetic characteristics of DFSP. Typically DFSP is CD34 positive and factor XIIIa negative, allowing its differentiation from dermatofibroma.

How well did you know this?
1
Not at all
2
3
4
5
Perfectly
73
Q

74- Amplification of which of the following genes is associated with Merkel cell carcinoma?

A. L-Myc
B. C-Myc
C. GLI1
D. CDKN2A
E. PTEN

A

►A

The L-Myc gene has been found to be amplified in Merkel cell carcinoma but not in normal skin. C-Myc has been found to be amplified in neuroblastoma. GLI1 is a transcription factor involved in hedgehog signaling and a potential target in basal cell carcinomas. CDKN2A is implicated in familial forms of melanoma that are associated with pancreatic cancer. PTEN is a tumor suppressor gene that is mutated in Cowden Disease, Bannayan-Riley-Ruvalcaba Syndrome, and Proteus Syndrome.

How well did you know this?
1
Not at all
2
3
4
5
Perfectly
74
Q

75- The most common location for a basal cell carcinoma is:

A. Lower eyelid
B. Forehead
C. Ears
D. Back
E. Shoulders

A

►A

The most common location for a BCC is the lower eyelid. It is the most common epithelial tumor of the eyelid.

How well did you know this?
1
Not at all
2
3
4
5
Perfectly
75
Q

76- If left untreated, which of the following is not at risk for malignant transformation?

A. Bowenoid papulosis
B. Cutaneous horn

C. Actinic cheilitis
D. Leukoplakia
E. Stucco keratosis

A

►E

Cutaneous horn can overlie an AK or SCC or a benign lesion and it presents as a conical protuberance arising from an erythematous base. Actinic cheilitis results from the confluence of multiple AKs on the lips. Leukoplakia is a clinical diagnosis and is defined as a white patch in the oral cavity. It is the most common premalignant condition of the oral cavity. Bowenoid papulosis manifests clinially as multiple red-brown warty papules that histologically represent high grade squamous intraepithelial lesions.

How well did you know this?
1
Not at all
2
3
4
5
Perfectly
76
Q

77- Currently, the surgical margin for melanomas that measure less than 2 mm in thickness is:

A. 1 mm
B. 0.5 cm
C. 1 cm
D. 2 cms
E. 3 cms

A

►C

The current surgical margins are 0.5 cms for melanoma in situ, 1 cm for melanomas that measure less than 2mm in thickness and 2cm for melanomas >2mm.

How well did you know this?
1
Not at all
2
3
4
5
Perfectly
77
Q

78- Denileukin diftitox (ONTAK®):

A. is a systemic treatment option for CTCL
B. is a diphtheria fusion toxin
C. targets the interleukin-2 receptor
D. None of these answers are correct
E. All of these answers are correct

A

►E

Denileukin diftitox is as diphtheria fusion toxin that targets the IL-2 receptor. It is a systemic treatment alternative for recalcitrant or advance CTCL.

How well did you know this?
1
Not at all
2
3
4
5
Perfectly
78
Q

79- Maple leaf-like structures seen on dermoscopy are characteristic of which lesion?

A. Seborrheic keratoses
B. Pigmented basal cell carcinoma
C. Dermal nevi
D. Melanoma
E. Hemangioma

A

►B

Maple leaf-like structures seen on dermoscopy are an important diagnostic criterion for pigmented basal cell carcinomas. They correspond to the heavily pigmented cells in the bas aloid cells.

How well did you know this?
1
Not at all
2
3
4
5
Perfectly
79
Q

80- Which one of the following agents has demonstrated potential benefit as a chemopreventive to UV-induced skin cancer?

A. Prostaglandin E2
B. Vitamin D
C. Arachidonic acid
D. Celecoxib
E. Vitamin E

A

►D

Cyclooxygenase-1 and -2 and enzymes that catalyze the conversion of arachidonic acid to prostaglandins. It is believed the prostaglandin E2 (PGE2), whose levels are increased by ultraviolet irradiation, is pro-inflammatory and may contribute to skin carcinogenesis. In a study by Orengo et.al., hairless mice who were given celecoxib were found to have a significantly longer latency period between exposure to ultraviolet light and the development of skin carcinomas.

How well did you know this?
1
Not at all
2
3
4
5
Perfectly
80
Q

81- UVB induced mutations on the PTCH gene is associated with the development of:

A. BCC
B. Merkel cell carcinoma
C. Angiosarcoma
D. BCC and Merkel cell carcinoma
E. Merkel cell carcinoma and Angiosarcoma

A

►A

The p53 and PTCH genes are the major targets of UVB for the development of BCC. Other genes involved include the Smoothened-activating mutations, and PTCH2 mutations.

How well did you know this?
1
Not at all
2
3
4
5
Perfectly
81
Q

82- Periungual Squamous cell carcinoma has been linked to which HPV type(s)?

A. 6, 11
B. 2, 4
C. 16
D. 13
E. 8

A

►C

Infections associated with Human Papilloma Virus can produce growths on the epithelial or mucosal surfaces. There are over 100 strains of these viruses, and some of these strains can predispose to intraepithelial carcinomas, particularly when involving the anal or g enital mucosa. In general, HPV strains 16 and 18 are classified as his risk virus types and can be associated with cervical cancer , oral cancer, anal cancer and periungual cancers. HPV 6 and 11 are associated with condyloma acuminata; HPV types 2, 4 are associated with common warts; HPV type 13 has been associated with Heck’s disease and HPV type 8 has been associated with epidermal dysplasia verruciformis.

How well did you know this?
1
Not at all
2
3
4
5
Perfectly
82
Q

83- Which of the following features of thin melanomas (<1 mm thick) has not been associated with an increased risk of metastasis?

A. Regression
B. Location
C. Age
D. Gender
E. P53 expression

A

►E

Regression in malignant melanoma appears histologically as a focal area of fibrosis with lymphocytes and melanophages in the papillary dermis. The presence of regression precludes accurate measurement of true thickness of the melanoma. Extensive regression in thin melanomas has a significant association for the risk of metastases.

How well did you know this?
1
Not at all
2
3
4
5
Perfectly
83
Q

84- What is the most common site of metastasis for this dermal tumor?

A. Lung
B. Brain
C. Kidney
D. Liver
E. Bone

A

►A

Dermatofibrosarcoma protuberans is a rare, low-grade dermal sarcoma. Typically, the lesion occurs as a painless subcutaneous mass that grows slowly. This malignancy typically has lateral spread but invade deep. Metastasis is rare but has been reported to the lung.

How well did you know this?
1
Not at all
2
3
4
5
Perfectly
84
Q

85- Which substance does p53 normally activate to promote apoptosis via inhibition of bcl-2?

A. p21
B. p16
C. Puma
D. Mdm2
E. Akt

A

►C

p53 is the most commonly mutated tumor suppressor gene involved in human cancer and is often mutated in SCC and BCC. p53 acts via two main pathways, 1) activation of p21 (Cdk inhibitor) which leads to cell cycle arrest, and 2) activation of Puma which inhibits Bcl-2 (apoptosis inhibitor) thereby leading to cell death. CKDN2A, a gene that when mutated leads to a risk of melanoma, acts via 1) activation of p16 (another Cdk inhibitor) and 2) activation of p14ARF which inhibits Mdm2 (which normally degrades p53). Akt is involved in the PI3K-Akt signaling pathway and inhibits cell cycle arrest and apoptosis.

How well did you know this?
1
Not at all
2
3
4
5
Perfectly
85
Q

86- The most common location for primary mucinous carcinoma is:

A. neck
B. eyelid
C. areola
D. scrotum
E. nose

A

►B

Mucinous carcinoma presents as a slowly growing, asymptomatic, round, erythematous nodule on the head and neck. Forty percent of cases occur on the eyelid. Histologically it is characterized by the presence of large areas of mucin (―sea of mucousǁ) with small islands of basophilic epithelial cells. Primary mucinous carcinoma of the skin has an indolent course. Local recurrence occurs in 1/3 of patients following excision. The rate of metastasis is low (9.6%).

How well did you know this?
1
Not at all
2
3
4
5
Perfectly
86
Q

87- All of the following disorders may manifest as exfoliative dermatitis except

A. Behcet’s disease
B. Psoriasis
C. Pemphigus foliaceus
D. Drug reaction
E. Sezary syndrome

A

►A

The image shows erythrodermic patient with generalized desquamation of skin. Skin biopsy and blood tests the diagnosis of Sezary syndrome. Sezary syndrome is a leukemic variant of mycosis fungoides, is characterized by the triad of pruritic erythroderma, g eneralized lymphadenopathy, and the presence of Sezary cells more than 1,000 cell/mm3 (abnormal, large hyperconvoluted lymphocytes) in peripheral blood . Other manifestations include scaling and fissuring of palms and soles, alopecia, pruritus, peripheral edema, and nail dystrophy. Exfoliative dermatits can be manifested in many conditions other than Sezary syndrome inculude pemphigus foliaceus, psoriasis and drug reaction.

87
Q

88- The human papilloma virus type associated with red brown smooth and warty papules is

A. HPV 1
B. HPV 5
C. HPV 7
D. HPV 13
E. HPV 16

A

►E

Bowenoid papulosis manifests clinically as multiple red-brown warty papules or confluent planques on the external genitalia. These lesions may resemble genital warts but histologically
represent high-grade squamous intraepithelial lesions. Bowenoid papulosis is caused by infection by HPV and linked to HPV 16, 18, 31, 35, and 39.

88
Q

89- A patient has a malignant melanoma 1.6mm thick with ulceration and a micrometastasis in 1 node. The patient’s staging according to the American Joint Committee on Cancer Staging System is:
A. IIC
B. IIIA
C. IIIB
D. IIIC
E. IV

A

►C

An ulcerated tumor of any size with micrometastasis in 1 node is T1-4b N1a M0. This corresponds to stage IIIB.

89
Q

90- The type #1 site of metastasis for dermatofibrosarcoma protuberans is:

A. Lungs
B. Kidney
C. Stomach
D. Bladder
E. Colon

A

►A

The #1 site of metastasis for DFSP are lungs. It has a low grade of metastasis <5% and treatment is wide local excision with 3 cm margins.

90
Q

92- Immunohistochemistry of dermatofibrosarcoma protuberans typically reveals:

A. CD34 negative and factor XIIIa positive
B. CD34 negative and factor XIIIa negative
C. CD34 positive and factor XIIIa positive
D. CD34 positive and factor XIIIa negative
E. None of these answers are correct

A

►D

Typically DFSP is CD34 positive and factor XIIIa negative, allowing its differentiation from dermatofibroma.

91
Q

93- All of the following statements are true regarding angiosarcomas EXCEPT:

A. They occur more commonly in Caucasians than in non-Caucasians
B. Men are more often affected than women
C. They are rarely symptomatic
D. Radiation is usually employed after surgical excision
E. Cervical lymph nodes are a common site of metastases

A

►C

Angiosarcomas are very rare, aggressive vascular tumors. They occur most commonly in the head and neck region of white, elderly individuals. Men are more commonly diagnosed with this neoplasm. The lesion initially arises as a painless, purple macule or plaque on the scalp or face. Later on it becomes an elevated bluish or purple nodule that may ulcerate. Common symptoms include bleeding, edema, and ultimately pain. Cervical lymph node and hematogenous metastases commonly occur. Wide surgical excision is the treatment of choice, with radiation therapy usually employed after surgical excision.

92
Q

94- Which of the following are true for the sign of Leser-Trelat?

A. Lesions are commonly located on the chest and back
B. Classically described to appear in a ―Christmas treeǁ pattern
C. Usually associated with internal malignancies

D. Eruptive nature
E. All of these answers are correct

A

►E

Characterized by the sudden appearance of multiple SKs, the sign of Leser-Trelat may have lesions located anywhere in the body. However, the eruptionǁ is classically described to appear in a Christmas treeǁ pattern commonly located on the chest and back. It may be associated with internal malignancies such as adenocarcinomas.

93
Q

95- The risk of metastasis from SCC increases with:

A. Tumor size
B. Depth of invasion
C. Degree of differentiation
D. Immunosupression
E. All of these answers are correct

A

►E

Tumor size (greater than 2 cms), location (lips, ears, eyelids), depth of invasion, degree of differentiation, perineural invasion, immunosupression, as well as recurrent tumors and those that arise in areas of chronic inflammation are all factors that increase the risk of metastasis.

94
Q

96- All of the following statements regarding Bowen’s disease are true EXCEPT:

A. Lesions arising on the lower limbs are more common in men than in women
B. The basement membrane remains intact on histopathology
C. 5% of patients with Bowen’s disease develop invasive squamous cell carcinoma
D. Chronic sun exposure is a risk factor for Bowen’s disease
E. Mucosal Bowen’s disease may appear as a verrucous plaque

A

►A

The most common locations for BD include the head and neck regions and the extremities. BD arising on the lower limbs is frequently found in women, whereas lesions located on the ears and scalp are more common in men. The basement membrane remains intact on histopathology. 5% of patients develop invasive SCC. Risk factors include chronic sun exposure, immunosuppression, HPV, arsenic exposure, and ionizing radiation. Mucosal BD may appear as a verrucous or polypoid plaque, as an erythroplakic patch, or as a velvety red plaque.

95
Q

97 -The incidence of cutaneous squamous cell carcinoma in organ transplant recipients is increased by how much compared with the general population?

A. 2 fold
B. 5 fold
C. 10 fold
D. 20 fold
E. 65 fold

A

►E

The risk of skin cancer in organ transplant patients is dramatically increase and may be more aggressive. In one study, SCCǁs had a 65x increased incidence, BCC 10x, and melanoma 3.4x.

96
Q

98- What is the most common location for pagetoid reticulosis?

A. Trunk
B. Head and neck
C. Hands and feet
D. Flexural sites of upper and lower extremities
E. Genitals

A

►C

Pagetoid reticulosis is an indolent cutaneous T-cell lymphoma. Pagetoid reticulosis favors an acral distribution and typically presents as scaly oval plaques. Epidermotropism is present, with tumor

cells being CD4 positive or CD8 positive. Another interesting immunohistochemical finding is the absence of CD45 expression.

97
Q

99- A patient presents with tender papules with a pseudo-Darier’s sign. She has other family members with the same condition. Screening should be performed to rule out which malignancy?

A. Gastric carcinoma
B. Ovarian carcinoma
C. Renal cancer
D. Testicular cancer
E. Lung cancer

A

►C

Reed’s syndrome is an autosomal dominant disease with incomplete penetrance characterized by uterine and cutaneous leiomyomas. Clinically, leiomyomas appear as flesh colored or pinkish brown dermal papules or nodules that range from 0.2 to 2.0 cm in diameter. Their presentation may otherwise be quite variable. They may be isolated or many in number, may be variably distributed or dermatomal, and may be asymptomatic or painful in response to pressure or cold. The predisposition gene for Reedǁs syndrome has been localized to chromosome 1q42.3-43 and the gene encoding fumarate hydratase. Currently, the United States National cancer Institute is recommending screening for all patients with leiomyomatosis to evaluate for an occult renal malignancy.

98
Q

100- All of the following cytokines have demonstrated therapeutic benefit in the treatment of melanoma EXCEPT:

A. IFN-alpha
B. IL-2
C. TNF-alpha
D. IL-10
E. GM-CSF

A

►D

IFN-alpha, IL-2, TNF-alpha, have all been demonstrated to have some therapeutic benefit in t he treatment of melanoma. GM-CSF is the immune stimulator in the vaccine Talimogene laherparepvec. IL-10 has been used to treat inflammatory disorders such as atopic dermatitis and psoriasis.

99
Q

101- A 60 year-old female presents with a well-demarcated, scaly, erythematous plaque on her right shin. The biopsy shows full thickness epidermal atypia with scattered mitotic figures and overlying parakeratosis. Howerver, the basement membrane remains intact. According to the aforementioned information, the diagnosis is:

A. BCC
B. SCC
C. Bowen’s disease
D. Angiosarcoma
E. CTCL

A

►C

Bowenǁs disease arising on the lower limbs is frequently found in women; whereas lesions located on the ears and scalp are more common in men. The epidermal dysplasia does not interrupt the basement membrane confirming the diagnosis of SCC in situ.

100
Q

102- What would be the characteristic histopathologic findings of this lesion?

A. Full thickness keratinocytic atypia
B. Cornoid lamella
C. Pale staining cells
D. Horn pseudocysts

E. Wedge shaped granular layer with lichenoid infiltrate

A

►B

Five clinical variants of porokeratosis are recognized. They are the classic porokeratosis of Mibelli, disseminated superficial actinic porokeratosis, prokeratosis palmaris et plantaris disseminata, linear porokeratosis, and punctate porokeratosis. On histopathology they have varying degrees of a cornoid lamella. This appears as a column of parakeratosis.

101
Q

103- Topical 5-Fluorouracil:

A. Interferes with the synthesis of DNA and RNA
B. Is an alternative for the treatment of actinic keratosis
C. May cause pruritus and burning at the site of application
D. All of these answers are correct
E. None of these answers are correct

A

►D

Topical 5-FU blocks the methylation reaction of deoxyuridylic acid to thymidylic acid, thus interfering with the synthesis of DNA and RNA. Normal side effects during treatment include pruritus and burning at the site of application. It is a treatment option for patients with multiple AKs within an area.

102
Q

104- Knowing that the patient in this picture has a biopsy confirmed BCC, which of the following would be the treatment of choice:

A. Conventional excision
B. Imiquimod
C. Radiation
D. Mohs micrographic surgery
E. Cryosurgery

A

►D

Providing the highest cure rates, Mohs micrographic surgery is the treatment of choice for large BCCs (1cm on the face or 2 cms on the trunk), located on high-risk anatomic areas (ear, eyelids, lips, genitals, nose, temples). , Morpheaform BCCs, tumors with positive margins after conventional excision, and recurrent BCCs, are also an indication for Mohs surgery.

103
Q

105- Mutations of the p53 gene has been associated with the development of:

A. Melanoma
B. Kaposi’s sarcoma
C. Actinic keratosis
D. Molluscum contagiosum
E. All of these answers are correct

A

►C

UVB radiation triggers the formation of thymidine dimers both in DNA and RNA, resulting in mutated keratinocytes. The mutations occur on the tumor suppressor gene p53 within the keratinocytes resulting in impairment of the mechanism of apoptosis. Therefore, clonal expansion of mutated keratinocytes may occur leading to the formation of AKs.

104
Q

106 -Spitz nevi can appear during childhood. The risk factors for metastatic Spitz include all of the following except:

A. Ulceration
B. Increased Breslow thickness
C. Increased mitoses

D. More H-RAS mutations
E. Location on the back

A

►E

Features of atypical spitz include location in the back over one cm and can have irregular color and border. The risk factors for metastatic Spitz is ulceration, increased Breslow thickness, atypical mitotic features and more H-RAS mutations as opposed to BRAF and NRAS.

105
Q

107- Histologically, this lesion is shows plump, polygonal cells arranged in nests and fascicles with granular cytoplasm. Which immunohistochemical stain would be positive?

A. Colloidal iron
B. Von kossa
C. Warthin-starry
D. S-100
E. CD68

A

►D

Granular cell tumors are benign growths which typically occur on the tongue. They are typically well-circumscribed, raised, firm nodules. Histologically, the cells have uniform nuclear characteristics and granular cytoplasm due to presence of lysozyme. The lesions are PAS positive and S-100 positive.

106
Q

108- The most common site for intra oral melanoma is?

A. buccal
B. soft palate
C. hard palate
D. gingiva
E. uvula

A

►C

Multiple case series have shown that the hard palate (and specifically the anterior hard palate/ alveolar arch) is the highest risk location for intra-oral melanoma. Prognosis is generally
worse for oral melanoma, which is usually due to delay in diagnosis and presentation with more invasive disease.

107
Q

109- Which of the following variants of mycosis fungoides is best diagnosed using a punch biopsy instead of a broad superficial shave biopsy?

A. Woringer-Kolopp pagetoid reticulosis
B. Syringotropic mycosis fungoides
C. Ketron-Goodman pagetoid reticulosis
D. Poikilodermatous mycosis fungoides
E. Sezary syndrome

A

►B

Syringotropic and folliculotropic mycosis fungoides (MF) are the variants of MF that should be diagnosed by punch biopsy. Both variants of pagetoid reticulosis, Woringer-Kolopp disease and the disseminated Ketron-Goodman disease, are best diagnosed with a broad shave biopsy. Classic and poikilodermatous MF should also be diagnosed with a shave biopsy. To rule out Sezary syndrome, flow cytometry should be performed.

108
Q

110- Sentinel lymph node biopsy in malignant melanoma:

A. Has gained acceptance for the treatment of MM of intermediate thickness (14mm)
B. Is mainly being used for a more accurate staging
C. Is not recommended
D. None of these answers are correct
E. Has gained acceptance for the work up of MM of intermediate thickness (14mm) and is mainly being used for a more accurate staging

A

►E

Sentinel lymph node biopsy has gained acceptance as part of the work up for melanomas 1-4 mm in thickness. Even though it has been widely accepted due to its low morbidity and high feasibility, the role of SLN biopsy in survival has not been established. It is mainly being used for a more accurate staging, prognosis and to determine if further adjuvant therapy is necessary.

109
Q

111- Which of the following immunohistochemistry marker is negative in angiosarcomas?

A. CD31
B. CEA
C. Cytokeratin
D. CD34
E. Factor VIII

A

►B

Angiosarcomas are CD31, CD34, factor VIII, and cytokeratin positive. The carcinoembryonic antigen (CEA) stains positive in Paget‘s disease, metastatic adenocarcinoma, and tumors with eccrine differentiation.

110
Q

112- Which of the following melanoma scenarios have the best prognosis?

A. Twenty-one year old female with primary lesion located on the right lower leg
B. Twenty-one year old male with primary lesion located on the chest
C. Thirty-six year old male with primary lesion located on the back
D. Twenty-one year old female with primary, ulcerated lesion located on the right lower leg
E. Thirty-six year old male with primary lesion located on his left lower leg with palpable inguinal lymph nodes

A

►A

Increasing age and male gender have a negative effect on survival. Primary lesions located on the extremities have a better prognosis than those located on the trunk, head or neck region. Ulceration is also considered a poor prognostic factor. Furthermore, there is a significantly lower survival for

those patients with palpable metastatic nodes (macrometastasis) when compared to those with micrometastatic nodes (nonpalpable).

111
Q

113- What is the most common location of metastasis of basal carcinoma?

A. liver
B. lung
C. bone
D. lymph node
E. skin

A

►D

Size of a BCC is related to its risk for metastasis. 3cm BCC has a 1.9% risk of metastasis. If a BCC is >10cm it has approximately a 50% risk of metastasis. If metastasis does occur it most commonly metastasizes to lymph nodes>lung>bone>skin>liver.

112
Q

114 -An elderly white man presents with a slowly enlarging, well-demarcated pink, scaly plaque on the neck. Biopsy of the lesion reveals epidermal dysplasia and keratinocytic disorganization with preservation of the basement membrane. Hyperkeratosis and parakeratosis are also present, and numerous atypical keratinocytes are seen throughout the epidermis, with loss of polarity, atypia, and mitoses. A chronic inflammatory infiltrate is present in the upper dermis. Which of the following treatment(s) should be considered for this patient?

A. Conventional excision
B. Mohs micrographic surgery
C. Imiquimod
D. Conventional excision and Mohs micrographic surgery
E. All of these answers are correct

A

►D

The patient has Bowen’s Disease, or squamous cell carcinoma in situ. Multiple treatment options are available, including conventional excision, Mohs micrographic surgery, imiquimod,

cryosurgery, and 5-FU, among others. Conventional excision is the most commonly used method. With this method, not only is the lesion removed, but it also provides the specimen for histologic verification to rule out invasive SCC. Recurrence rates of 5% have been reported with conventional excision. Mohs micrographic surgery is an option when the lesion is located in areas where there is an increased risk of sub-clinical spreading, or when tissue sparing is a priority. Imiquimod is not approved to treat SCCIS.

113
Q

115- Compared with the general population, what is the overall risk of developing cutaneous and systemic malignancies in organ transplant recipient?

A. 4 fold
B. 10 fold
C. 25 fold
D. 50 fold
E. 100 fold

A

►A

Organ transplant recipients are at increased risk of having both systemic and cutaneous develop. The estimated increased risk is 3-4 times that of the general population. However, the risk of skin cancer alone is much higher (e.g. SCC ― 65x increase, BCC 10-fold increase, Melanoma 3.4x increase). This increased risk is thought to be partially due to the immunosuppressant agents used to prevent graft rejection.

114
Q

116- When performing a biopsy of a suspected keratoacanthoma:

A. Fusiform incision through the entire KA may be performed
B. It is necessary to biopsy down to subcutaneous fat
C. A full-thickness shave biopsy is acceptable
D. A complete excisional biopsy may be performed
E. All of these answers are correct

A

►E

When considering a keratoacanthoma, it is important to obtain a biopsy of the specimen down to the subcutaneous fat. This can be achieved either by complete excisional biopsy, full-thickness shave biopsy, or fusiform incision through the entire KA including its center and sides.

115
Q

117- Merkel cell carcinoma should be treated with what size surgical margins?

A. 2mm
B. 5mm
C. 1cm
D. 3cm
E. 5cm

A

►D

Merkel cell carcinoma is an aggressive rare tumor of the skin accounting for less than 1% of cutaneous malignancies. Also known as neuroendocrine cancer of the skin, this tumor presents as a painless red to violaceous, firm, solitary, nodule that usually presents on sun exposed areas such as the head, neck and upper extremities. These tumors present usually during the 6th and 7th decades, and have a 2 year survival rate of 50-70%. Because of this tumor’s high potential for regional and distal metastasis, this tumor should be excised with wide local excision with 3cm surgical margins.

116
Q

118- Which of the following is a clinical stage of keratoacanthomas?

A. Plaque
B. Mature
C. Proliferative
D. Resolving
E. All of these answers are correct except plaque

A

►E

Keratoacanthomas have three, consecutive, clinical stages: proliferative, mature and resolving. The proliferative stage is characterized by the appearance of a rapid growing papule. This phase is followed by the mature stage when the lesion acquires its ch aracterisitic dome-shaped appearance

with a central, keratinous core. Tumor resorption occurs during the involution stage resulting in a slightly depressed, hypopigmented scar.

117
Q

119- All of the following are risk factors for metastasis from a primary squamous cell carcinoma EXCEPT:

A. Increasing tumor size
B. Location on the ear
C. Recurrent tumor
D. Tumor within ulcer
E. All of these answers are correct

A

►E

The risk of metastasis from a primary SCC increases with tumor size (>2cm), location (lips, ear, and eyelid, among others), depth of invasion, immunosuppression, degree of differentiation, and perineural invasion. Recurrent tumors and SCCs arising in areas of chronic inflammation (such as osteomyelitis, burn scars, or ulcers) are also considered high risk for metastases.

118
Q

Which of the following lesions are ǁbetter felt than seenǁ:

A. Seborrheic keratosis
B. SCC in situ
C. Actinic keratosis
D. Sebaceous hyperplasia
E. All of these answers are correct

A

►C

Typical AKs appear as flat, erythematous, rough papules that are better felt than seen.

119
Q

121- What is the eponym for this metastasis to the umbilicus?

A. Tripe palm
B. Sign of Leser-Trelat
C. Pityriasis rotunda
D. Trousseau syndrome
E. Sister Mary Joseph Nodule

A

►E

Sister Mary Joseph nodule is a cutaneous metastasis that is most commonly associat ed with stomach, large bowel, ovary, and pancreatic cancer. Tripe palms are most commonly associated with lung, Leser-Trelat with adenocarcinomas of the stomach, colon and breast, Trousseau Sign with carcinoma of the pancreas and pityriasis rotunda with hepatocellular carcinoma and gastric cancer.

120
Q

122 -On histologic examination of the adenocarcinoma of the perineal area, which of the following stains would NOT be positive?

A. PAS
B. Mucicarmine
C. CEA
D. EMA
E. HMB45

A

►E

Extramammary Paget’s cutaneous adenocarcinoma of glandular differentiation. Approximately 25% of these tumors are associated underlying neoplasms. Histologically, the Paget cells stain positively with PAS, mucicarmine, CEA, EMA, LMW keratin. They are HMB45 negative which is a melanocytic marker.

121
Q

123 -The cure rate of cryotherapy as a treatment for actinic keratoses is:

A. 80%
B. 85%
C. 90%
D. 95%
E. 99%

A

►E

Cryotherapy is the most common treatment for AKs, with a cure rate of 98.8%.

122
Q

124- Which of the following is true regarding poikilodermatous mycosis fungoides?

A. Majority of cases are predominantly CD8(+)
B. Patients typically have a later age of onset compared to classic mycosis fungiodes
C. Can be associated with LyP
D. More women than men affected
E. Patients typically do not respond well to phototherapy

A

►C

While there is an overrepresentation of CD8+ cases compared to classic mycosis fungoides, more cases are still predominantly CD4+. Patients typically present at a younger age (median age 44 years), with a slight male predominance. There is an excellent response to photo therapy with clinical improvement in ~90% of patients treated. There is an increased association with LyP compared to other types of mycosis fungoides. (Abbott et al, JAAD 2011; in press)

123
Q

125- The most common locations of microcystic adnexal carcinoma include all of the following, except:

A. Perioral
B. Nasolabial
C. Trunk

D. Periorbital
E. Perioral,nasolabial, and periorbital

A

►C

Microcystic adnexal carcinomas are more commonly located in the perioral, nasolabial, or periorbital areas.

124
Q

126- Merkel cell carcinoma can be summarizes with certain clinical features. All of the following are features except:

A. Asymptomatic
B. Expanding rapidly
C. Older than 50
D. Older than 60
E. Ultraviolet-exposed area in person with fair skin

A

►D

Merkel cell carcinoma can be summarized with “AEIOU” that stands for expanding rapidly, immune suppression, older than 50 years, ultraviolet-exposed area in person with fair skin.

125
Q

127- In which of the following ethnic groups is squamous cell carcinoma the most common type of skin cancer?

A. Asian Indians
B. Caucasians
C. Hispanics
D. Japanese
E. Chinese

A

►A

SCC is the most common type of skin cancer in Asian Indians and Blacks. BCC is the most common type of skin cancer in Caucasians, Japanese, Chinese, and Hispanic people.

126
Q

128- All of the following are true of melanoma-associated leukoderma except:

A. Lesions resemble vitiligo
B. Portends a worse prognosis
C. Seen in patients with metastatic disease but no primary lesion
D. Histology resembles that of a halo nevus
E. Lesions develop distant to melanoma

A

►B

Hypomelanosis associated with melanoma most commonly may be seen in three ways. The first type is analogous to a halo nevus. The second is a remote leukoderma distant from the primary lesion. The third is a vitiligo like leukoderma. It may portend a better prognosis in compa rison to others with the same stage of disease.

127
Q

129- Which of the following is commony seen in seborrheic keratoses when examined with dermoscopy?

A. Maroon lagoons
B. Network
C. Pigment globules
D. Milia like cysts
E. Blue-gray veil

A

►D

The main dermoscopic features of seborrheic keratoses include comedolike openings, fissures, milialike cysts, fingerprinting and lack of true pigment network. In fact, it is the lack of true pigment network, branched streaks, and pigment globules that differentiate SKs from melanocytic lesions.

128
Q

130- A 60 y/o male smoker with multiple sclerosis presents for evaluation of a painful 5cm x 4cm plaque on right posterior shoulder, which appeared over 3 months. What is the most likely diagnosis?

A. Irritated Seborrheic Keratosis
B. Squamous Cell Carcinoma
C. Basal Cell Carcinoma
D. Metastatic Carcinoma
E. Malignant Melanoma

A

►C

Basal cell carcinoma. It has a male to female ratio 2:1. UVB damages DNA and produces a C -T transition mutation (65% of BCC). P53 and PTCH genes are commonly affected. It has a slow, indolent growth potential: metastatic potential 0.0028% to 0.1%.; known to metastasize to lymph nodes and lungs.

129
Q

131- An elderly man presents to your office with a telangiectatic, violaceous 1cm dome-shaped nodule on the neck. Biopsy reveals large, solid nests of cells of intermediate size, with a trabecular pattern at the periphery. These cells involve the dermis and spread into the subcutaneous fat, but spare the overlying epidermis. The cells are round and uniform in size, with a round to oval nucleus, small nucleoli, and evenly dispersed chromatin. Numerous mitotic figures and necrotic areas are seen. Neuron specific enolase is positive. Which of the following is true regarding this patient’s diagnosis?

A. Mohs micrographic surgery is contraindicated in treatment of this lesion
B. Vimentin and desmin stains may be positive
C. S-100 stains should be positive
D. This lesion may contain ACTH
E. The neoplasm should not contain gastrin

A

►D

This patient has a Merkel cell carcinoma. Vimentin, desmin, and S-100 are consistently absent in MCC. Mohs micrographic surgery has been used successfully for the treatment of MCC, with or

without adjuvant therapy. This neoplasm may sometimes contain several neuropeptides including vasoactive intestinal peptide, calcitonin, ACTH, gastrin, and somatostatin.

130
Q

132- The translocation for dermatofibrosarcoma protuberans is:

A. Chr 17 and 22
B. Chr 7 and 10
C. Chr 9 and 13
D. Chr 20 and 2
E. Chr 29 and 30

A

►A

The translocation for DFSP is Chr 17 and 22. These chromosomes involve the fusion of PDGF beta chain and collagen type I alpha I genes. DFSP are generally CD34+ and factor XIIIa negative.

131
Q

133- What virus is most closely associated with these lesions in this HIV infected patient?

A. Human herpes virus 2
B. Human herpes virus 6
C. Human herpes virus 8
D. Cytomegalovirus
E. Ebstein Barr virus

A

►C

Kaposi’s sarcoma is an AIDS defining illness. It if found at a much higher incidence in homosexual men. Human herpes virus 8 is thought to be pathogenic in this tumor.

132
Q

134- What is the most likely diagnosis?
A. Acquired digital fibrokeratoma
B. Supernumery digit

C. Verruca
D. Infantile digital fibroma
E. Acrochordon

A

►A

The condition shown is an acquired digital fibrokeratoma. It is a pedunculated, acral lesion with collarette which is defined by the lack of nerve twigs and bone on pathologic exam.

133
Q

135- Dermoscopy would be the least helpful in evaluating which lesion?

A. Pigmented basal cell carcinoma
B. Congenital melanocytic nevus
C. Seborrheic keratosis
D. Amelanotic melanoma
E. Hemangioma

A

►D

Amelanotic melanoma are difficult to diagnosis with the naked eye and even with the dermoscope. Pigmented basal cell carcinomas may have a characteristic maple leaf pattern,
arborizing blood vessels, and /or blue-grey ovoid nests. Seborrheic keratoses often have comedolike openings and milia cysts. Hemangiomas typically have red lagoons.

134
Q

136- The most common location of Merkel cell carcinoma is:

A. Digits
B. Trunk
C. Genitalia
D. Extremities
E. Head and neck

A

►E

Merkel cell carcinoma usually presents as a violaceous, solitary, dome-shaped nodule, most commonly located on the head and neck region.

135
Q

137- A 40 year-old female patient presents with the following lesion (see image). The biopsy report shows duct like structures, tadpole structures within a fibrotic stroma. According to the aforementioned information, the most likely diagnosis is:

A. Microcystic adnexal carcinoma
B. Seborrheic keratosis
C. Dermatofibrosarcoma protuberans
D. Merkel cell carcinoma
E. None of these answers are correct

A

►A

One of the most common locations for MAC includes the perioral area. Histologically it presents with poorly demarcated tumor cells invading the dermal and subcutaneous tissue. Islands of basaloid keratinocytes, horn cysts and duct structures are also seen within a desmoplastic stroma.

136
Q

138- All of the following may increase the incidence of SCC, EXCEPT:

A. Sun exposure
B. Immunosuppression
C. Increasing age
D. Proximity to the equator
E. All of the following may increase the incidence of SCC

A

►E

The incidence of SCC increases with age, and it is 35 times higher in individuals older than 75 years of age when compared to ages 50-55. The incidence doubles for each 8 to 10 degree decline in latitude, therefore, individuals living closer to the equator have a greater risk.

137
Q

139- Which of the following statements regarding prognosis for malignant melanoma is true?

A. Increasing age has a positive effect on survival.
B. Patients with primary lesions located on the extremities (except acral lesions) have a worse prognosis than those with tumors located on the trunk.
C. There is a higher survival rate for patients with palpable metastatic nodes compared to those with micrometastatic nodal disease.
D. For stage IV disease, patients with non-visceral metastases (eg skin, subcutis, distant lymph nodes) have a better prognosis compared with those with visceral metastases.
E. Male gender has a positive effect on survival.

A

►D

Increasing age and male gender have a negative effect on survival in melanoma. Patients with primary lesions located on the extremities (except acral lesions) have a better prognosis than those with tumors located on the head, neck, or trunk. There is a significantly lower survival for patients with palpable metastatic nodes (macrometastasis) when compared to those with micrometastatic nodes. For stage IV disease, patients with non-visceral metastases (eg skin, subcutis, distant lymph nodes) have a better prognosis compared with those with visceral metastases.

138
Q

140- A child presents with a giant congenital nevus overlying the back of the skull, extending onto the shoulders. It is ~15% body surface area, sparing the face and anterior neck. Which test should be ordered?

A. A head CT
B. A head MRI
C. A skull plain film X-ray
D. A skin biopsy
E. A PET scan

A

►B

Giant congenital nevi overlying the spinal columna nd skull can be associated with neurocutaneous melanosis. Presenting symptoms include increased cranial pressure, spinal cord compression or leptomeningeal melanoma. A MRI should be performed in these children to ruleout CNS involvement. The other tests would not be as useful in this situation.

139
Q

What is the most common location for this rapidly growing tumor?

A. Head and neck
B. Chest
C. Back
D. Arms
E. Legs

A

►A

Merkel cell carcinoma a cutaneous neuroendocrine carcinoma that usually occurs on the head and neck. It is a high grade malignant tumor with a 5-year mortality rate of 30-64%. On histopathology, there are trabecular strands of basophilic cells that stain with a characteristic paranuclear dot pattern with cytokeratin 20.

140
Q

142 -This lesion is associated with which of the following:

A. HPV 5
B. HPV 11
C. HPV 4
D. HPV 7
E. HPV 1

A

►B

HPV 6 and 11 are most often associated with anogenital warts or condyloma acuminata. They can present as sessile lesions on the skin or peducnulated cauliflower lesions. HPV type 5 is associated with epidermodysplasia verruciformis, HPV 4 is assoicatd with common warts, HPV type 7 is associated with Butcher warts, and HPV type 1 is associated with common warts.

141
Q

143- A 50 year-old female patient presents with a reddish indurated plaque located on her right clavicular area (see image). The biopsy shows the presence of monomorphic spindle cells in a storiform pattern, and deep subcutaneous infiltration. According to the aforementioned information, the most likely diagnosis is:

A. Microcystic adnexal carcinoma
B. Seborrheic keratosis
C. Dermatofibrosarcoma protuberans
D. Merkel cell carcinoma
E. None of these answers are correct

A

►C

DFSP intially arises as an asymptomatic, reddish or skin colored indurated plaque, which may later enlarge, and become raised, firm and multinodular. DFSP is histologically characterized by the presence of monomorphic spindle cells arranged in a “storiform” or “cartwheel” pattern.

142
Q

144- Dermoscopic features suggestive of malignant melanoma include:

A. Presence of 2 or more colors within the lesion
B. Blue-whitish veil
C. Asymmetric radial streaming
D. Abrupt interruption of pigment network in the periphery
E. All of these answers are correct

A

►E

Asymmetry, multicomponent pattern, blue-whitish veil, parallel-ridge pattern, atypical pigment network, uneven radial streaming, localized irregular and diffuse pigmentation, irregularly distributed globules, and regression structures are all dermoscopic features suggestive of malignant melanoma.

143
Q

145- Which of the following is the most important prognostic indicator in a patient with cutaneous lymphoma?

A. Age
B. Primary vs secondary cutaneous
C. Gender
D. Extent of cutaneous involvement
E. Subtype of lymphoma

A

►B

When staging lymphoma, it is critical to determine whether the lymphoma is primary cutaneous arising in the skin or secondary cutaneous arising in association with nodal or extranodal tumor. The prognosis is worse in secondary when compared to primary lymphomas, irrespective of histologic diagnosis.

144
Q

146- Which of the following immunologic drugs has been shown to increase survival of patients with stage III and stage IV melanoma?

A. Etanercept
B. Adalimumab
C. Ustekinumab
D. Ipilimumab
E. Infliximab

A

►D

A recent study in the New England Journal of Medicine reported an increase in survival of patients with stage III and stage IV melanoma with a new immunologic therapy agent named ipilimumab. The survival was increased from 6 months to 10 months (P < 0.001). All the other medicines are not used to treat melanoma; they are used to treat psoriasis, among other disease.

145
Q

147- The risk of a melanoma developing in a giant congenital melanocytic nevus is approximately:

A. 3%
B. 6%
C. 12%
D. 50%
E. Virtually all of these patients will develop melanoma

A

►B

Large/giant congenital nevi are greater than 20 cm or greater than 10% of the body surface area. The risk of melanoma in this type of lesion is approximately 6% by the age of 60 (Rhodes, 1981; Bett, 2005). 50% of these melanomas occur by 5 years of age.

146
Q

148- Which of the following is not considered a high-risk location of SCC:

A. Eyelids
B. Nose

C. Ear
D. Lips
E. Chest

A

►E

High risk locations for recurrence and metastases include the ―H-zoneǁ of the face, skin overlying cartilage and bony structures (e.g. preauricular area, retroauricular sulcus, nasolabial fold, inner canthus, philtrum, temple, upper lip, columella, nose, lower eyelid). Lesions located on the trunk and extremities are usually considered low risk unless aggressive histologic pattern is present.

147
Q

149- What type of skin cancer is associated with recessive dystrophic epidermolysis bullosa?

A. Squamous cell carcinoma
B. Basal cell carcinoma
C. Merkel cell carcinoma
D. Melanoma
E. Actinic keratosis

A

►A

Patients with recessive dystrophic epidermolysis bullosa havea high risk of aggressive SCC in the areas of chronic erosion and must be screened thoroughly and frequently. Actinic keratosis is not skin cancer.

148
Q

150- The mucin found in this lesion is produced by:

A. Fibroblasts
B. Keratinocyte
C. Merkel cells
D. Nail matrix
E. Osteoblast

A

►A

Digital mucous cysts are pseudocysts found on the dorsal digits between the distal interphalangeal joint and the nail fold. The fibroblasts in digital mucous cysts produce large quantities of hyaluronic acid.

149
Q

151- Which of the following are features of dermatofibromas, which help differentiate it from dermatofibrosarcoma protuberans?

A. Commonly located on the extremities
B. Factor XIIIa negative
C. Factor XIIIa positive
D. Commonly located on the trunk
E. Commonly located on the extremities and factor XIIIa positive

A

►E

Typically DFSP is CD34 positive and factor XIIIa negative, allowing its differentiation from dermatofibroma.

150
Q

152- What is the most common location for this lesion which lacks phosphorylase in the epidermal cells?

A. Back
B. Buttock
C. Leg
D. Neck
E. Sacrum

A

►C

Clear cell acanthomas are slow growing, benign growths which typically occur on the leg. They have a striking histologic appearance in the epidermis with sharp demaracation and enlarged, pale cells. Excess glycogen in the cells accounts for their clear appearance and is due to a defect in phosphorylase.

151
Q

153- The Gorlin syndrome is characterized by:

A. Multiple BCCs during childhood
B. Macrocephaly
C. Odontogenic keratocysts of the jaw
D. Autosomal-dominant inheritance pattern
E. All of these answers are correct

A

►E

Gorlins syndrome is characterized by the appearance of multiple BCCs during childhood, odontogenic keratocysts of the jaw, and skeletal defects. It is inherited in an autosomal dominant pattern.

152
Q

154- Seventy-five year old, male patient, with a 3cms pink, pearly nodule on his chest. You notice that the patient has a scar on the same area. The treatment of choice is:

A. Radiation
B. Conventional excision
C. Cryotherapy
D. Mohs surgery

E. Imiquimod

A

►D

The most likely diagnosis in this patient is recurrent BCC, therefore Mohs surgery is the treatment of choice. Furthermore, any NMSC measuring 3 cms located on any anatomical site is an indication for Mohs surgery.

153
Q

155- This type of keratoacanthoma is eruptive and disseminated in adulthood:

A. Grzybowski
B. Ferguson-Smith
C. KA centrifugum
D. Giant KA
E. Subungual

A

►A

Patients with Grzybowski keratoacanthoma have eruptive and disseminated KA during adulthood. KA centrifugum are large and solitary and Giant KA can invade underlying.

154
Q

156 -Which HPV serotype has been associated with periungual SCC?

A. 6
B. 16
C. 11
D. All of these answers are correct
E. HPV is not associated with SCC

A

►B

HPV infection has been associated with the development of cutaneous SCC. HPV 6 and 11 have been linked to genital lesions, whereas HPV 16 has been found in periungeal lesions.

155
Q

157- Keratoacanthomas have been linked etiologically to:

A. Ultraviolet exposure
B. Human papilloma virus
C. Chemical carcinogens such as tar and pitch
D. Smoking
E. All of these answers are correct

A

►E
The origin of KAs has not been established. Ultraviolet exposure, exposure to chemical carcinogens such as tar and pitch, as well as smoking, and a viral etiology, specifically the human papilloma virus, have all been proposed as etiologic factors.

156
Q

158- The treatment for Merkel cell carcinoma is all of the following except:

A. Wide local excision 2-3 cm
B. Mohs
C. Radiation sensitive therapy
D. Wide local excision 5-10 cm
E. Local excision of 1-2 cm

A

►E

Merkel cell carcinoma is a type of aggressive cancer with a high rate of recurrence. It is a neuroendocine carcinoma that is most commonly on sun exposed areas like the head and neck in the elderly. Treatment includes wide local excision of 2-3 cm, Mohs, radiation, and chemotherapy.

157
Q

159- Sezary syndrome:

A. Has characteristic Sezary cells in peripheral blood
B. The five year survival is estimated to be between 10-20%.
C. Is characterized by the triad of pruritic erythroderma, generalized lymphadenopathy, and presence of Sezary cells in peripheral blood
D. Is associated with a poor prognosis

E. All of these answers are correct

A

►E

Sezary syndrome is characterized by the triad of pruritic erythroderma, generalized lymphadenopathy, and presence of Sezary cells in peripheral blood. Sezary cells are abnormal, large hyperconvoluted lymphocytes. The five year survival is estimated to be between 10-20%.

158
Q

160- Which of the following best describes the incidence of skin cancer in transplant recipients from greatest to least?

A. BCC>melanoma>SCC>Merkel cell
B. SCC>BCC>melanoma>Merkel cell
C. BCC>SCC>melanoma>Merkel cell
D. SCC>BCC>Merkel cell>melanoma
E. Merkel cell>SCC>melanoma>BCC

A

►B

The incidence of skin cancer in transplant recipient (from greatest to least) is as follows: SCC>BCC>melanoma>Merkel cell.

159
Q

161- Histopathological evidence of epidermotropism may be seen in which of the following conditions:

A. Merkel cell carcinoma
B. SCC
C. CTCL
D. None of these answers are correct
E. All of these answers are correct

A

►C

Epidermotropism, defined as the presence of lymphocytes in the epidermis is characteristic of CTCL.

160
Q

162- 40 year-old male presented with a slow growing large tumor on the back. The patient undergoes wide excision with adjunctive chemotherapy. Which of the chemotherapeutic agents was used?

A. Imatinib
B. Cytarabine
C. Doxorubicin
D. Vincristine
E. Cisplatin

A

►A

Imatinib or Gleevec is used in the treatment of primary or locally recurrent dermatofib rosarcoma protuberans. Even with wide excision, the 5 year recurrence rate for this tumor can be 20-25%. Activation of the Platelet derived growth factor receptor associated with overexpression of Platelet Derived growth factor is central to the development of DFSP Imatinib is a protein tyosine kinase inhibitor for Bcr-Abl kinase and is used primary in chronic myelogenous leukemia with the Philadelphia chromosome defect. Imatinib also inhibits the typosine kinases associated with platelet derived growth factor and stem cell factor. Therefore, it directly inhibits the Platelet Derived Growth factor receptor signaling cascade,that plays a critical role in the pathogenesis and growth of DFSPs. While Imatinib has been successful in clinical trials, it is not yet FDA approved for the treatment of DFSPs. The other listed agents are not adjunctive or primary therapies for DFSP.

161
Q

163- Which site of squamous cell carcinoma has the greatest risk of metastasis?

A. lip
B. ear
C. eyelid
D. trunk
E. nose

A

►A

The lip has an approximate rate of metastasis of 13%, which is slightly higher than the metastasis rate of 11% from the ear. Risks for metastasis include size greater than 2cm, perineural invasion, immunosuppression, treatment history, degree of tumor differentiation, or location on the ear or lip.

162
Q
  • All of the following are true regarding curettage of giant congenital melanocytic nevi except:

A. Treatment remains a controversial topic
B. Best performed during the first 2 weeks of life
C. Decreases the risk of melanoma
D. Offers an adequate alternative to surgical excision
E. Mandates careful long-term follow-up

A

►C

Curettage of giant congenital nevis is a highly controversial treatment option. De Raeve and Roseeuw reported on 16 neonates who underwent curettage for treatment of their giant congenital nevi. They noted that it was best performed within the first two weeks of life. They claim that it offers an adequate cosmetic alternative to surgical excision. The risk of melanoma developing within the giant congenital nevus is not decreased and long-term follow-up is essential.

163
Q

165- What is the mean time-frame for development of Stewart-Treves Syndrome?

A. Less than 1 year
B. 1 year
C. 5 years
D. 10 years
E. >20 years

A

►E

Stewart-Treves syndrome refers to an angiosarcoma arising from chronic lymphedema. It typically occurs as a complication of long-lasting lymphedema of the arm after mastectomy and/or

radiotherapy for breast cancer. It is a very late complication of disease, usually developing several years later. The mean time-frame for development of Stewart-Treves Syndrome in the literature is 23 years (4-40 years).

164
Q

166- What kind of T-lymphocyte is the most common neoplastic cell in Cutaneous T-Cell Lymphoma:

A. CD4
B. CD8
C. Natural killer
D. None of these answers are correct
E. All of these answers are correct

A

►A

CTCL is a neoplasm of helper T cells (CD4) that originates in the skin.

165
Q

167- Commonly used immunohistochemical markers that are positive in Merkel cell carcinomas include all of the following EXCEPT:

A. Chromogranin A/B
B. Synaptophysin
C. Glial fibrillary acidic protein
D. Cytokeratin 20
E. Cytokeratin 8

A

►C

The most commonly used markers for MCC are monoclonal antibodies to cytokeratins 8, 18, and 20; neuron-specific enolase (the most constant marker); chromogranin A/B, and synaptophysin. Leukocyte common antigen, vimentin, desmin, glial fibrillary acidic protein, and S-100 are consistently absent in MCC.

166
Q

168- Which area of the face receives the most cumulative exposure to UV radiation?

A. Dorsum of nose
B. Orbital region
C. Base of the nose
D. Chin
E. Central cheek

A

►C

The most frequent sites of basal cell carcinomas in one study was the base of the nose (bordering the nasolabial fold and extending midway up the side of the nose), orbital region, apex of the nose, ear, forehead, temporal region, nasolabial area, and the buccal region (in order). The nose was by far the most frequent site of BCCǁs.

167
Q

169- Desmoplastic trichoepithelioma:

A. Is most common in middle-aged patients
B. More commonly appears in males than in females
C. Does not display foci of calcification or ossification
D. Presents as a well-circumscribed lesions located in the upper dermis
E. None of these answers are correct

A

►D

Desmoplastic trichoepithelioma is a variant of trichoepithelioma, an uncommon adnexal tumor with differentiation toward hair structures. It is three times more common in females and usually occurs in young adults. The tumor is a well-circumscribed nodule lesions located in the upper dermis. Strands or columns of basaloid cells are seen surrounded by fibrotic or desmoplastic stroma. Horn cysts may also be seen, as well as foci of sebaceous cells, calcification and ossification.

168
Q

170- Which of the following is true regarding digital HPV-associated squamous cell cancers?

A. The rate of metastasis approaches 15%.

B. HPV18 is the most common associated sybtype.
C. Mohs micrographic surgery yields a 20% recurrence rate.
D. Women outnumber men 2:1.
E. These lesions only occur in association with immunosuppression.

A

►C

According to Riddel et al (JAAD 2011;64(6):1147-1153), Mohs micrographic surgery, although the treatment of choice, results in a 20% recurrence rate, which is significantly higher than cutaneous SCC. HPV16 is most often implicated. Men outnumber women 2:1. The rate of metastasis averages between 2-3%. Although common in transplant patients, HPV associated digital SCCs can occur secondary to trauma and in immunocompetent patients.

169
Q

171- All of the following are true of thick melanomas (>3 mm) except:

A. Predominantly nodular type
B. Women affected more than men
C. Predilection for the head and neck
D. Mainly in older patients (>50 years)
E. Associated with fewer nevi

A

►B

According to a study performed by Chamberlain, et.al., thick melanomas (> 3 mm) were predominantly nodular in type. They occurred in older men, mostly on the head and neck and were associated with fewer nevi.

170
Q

172- Merkel cell carcinoma stains positively for:

A. Leukocyte common antigen
B. Neuron specific enolase
C. Vimentin
D. S-100
E. None of these answers are correct

A

►B

Merkel cell carcinoma is frequently diagnosed in individuals older than 50 years of age. However, there have been reports of MCC in children and young adults.

171
Q

173- Pseudorosettes in Merkel cell carcinoma:

A. Are seen in the trabecular variant
B. Are seen in the intermediate-cell type
C. Are seen in the small-cell type
D. All of these answers are correct
E. None of these answers are correct

A

►A

There are three histologic patterns of MCC: trabecular, intermediate-cell type, and small-cell type. The trabecular variant consists of interconnecting trabeculae separated by strands of connective tissue. Pseudorosettes may be seen in this type.

172
Q

174 -A patient was referred from the oncology service because of tumoral lesions of MF. There is no bone marrow involvement. What is the stage on this patient?

A. IIA
B. IIB
C. IIIA
D. IIIB
E. IV

A

►B

This patient has the tumor stage of mycosis fungoides(MF). Patients with tumoral lesions are automatically classified as IIB. IA is the limited patch or plaque disease with less than 10% BSA involved. IB is the generalized patch/plaque stage with more than 10%BSA but less than 80% BSA involved. IIA implies lymph node involvement. IIIA is erythroderma without blood

involvement. IIIB is erythroderma with low blood tumor burden. The stage IV of MF is divided in three substages: IVA1(high blood tumor burden-Sezary syndrome), IVA2(very abnormal nodes), IVB(visceral involvement).

173
Q

Which of the following is true regarding actinic keratoses?

A. Salicylic acid, tretinoin, and alpha-hydroxy acids are not useful in treatment
B. Low fat diets may decrease the incidence of AKs
C. Cure rates for treatment with topical 5-fluorouracil are higher than for treatment with cryotherapy
D. UVA from sunlight is most responsible for AK development
E. Male gender is not a risk factor for AKs.

A

►B

Salicylic acid, tretinoin, and alpha-hydroxy acids have been reported to be useful in treatment of AKs. Low fat diets have been reported to be useful in decreasing the incidence of AKs. Cryotherapy has a reported cure rate of 98.8% for AKs, whereas topical 5-FU has a reported cure rate of 93%. UVB, not UVA, is most responsible for AK development. Male gender is a risk factor for the development of AKs.

174
Q

176 -Which neoplasm is associated with the Stewart-Treves syndrome?

A. Renal leiomyomas

B. Basal cell carcinoma
C. Angiosarcoma
D. Keratoacanthoma
E. T cell lymphoma

A

►C

Stewart-Treves syndrome is the development of angiosarcoma in the setting of chronic lymphedema. Originally, named after radical mastectomy for the treatment of breast cancer. The term applies to the development of angiosarcoma in any chronic lymphedematous condition.

175
Q

177- The most common location for angiosarcoma is:

A. Legs
B. Arm
C. Head and neck
D. Trunk
E. Digits

A

►C

Angiosarcomas present as painless, purple nodules in the head and neck region of elderly individuals. Men are more frequently diagnosed with this aggressive neoplasm.

176
Q

178- A patient with multiple skin nodules has a biopsy suggestive of mycosis fungoides. The next appropriate step in making the diagnosis is:

A. Polymerase chain reaction
B. Complete blood count
C. Flow cytometry
D. Immunohistochemical stains
E. Chest xray

A

►A

In a patient with skin lesions and pathology suggestive of cutaneous lymphoma, gene rearrangement studies should be done to detect a clonal population of lymphocytes. Polymerase chain reaction (PCR) is approximately 1000 times more sensitive than Southern blot in the detection of a clonal population of cells. In addition, PCR may be used on formalin fixed tissue as well as fresh tissue, whereas Southern blot testing must be done of fresh tissue only.

177
Q

179- All of the following statements regarding Basal Cell Nevus Syndrome are true EXCEPT:

A. Associated tumors include medulloblastoma
B. It is inherited in an autosomal recessive manner
C. Hypertelorism is a feature
D. The affected gene is mutated in 30-40% of sporadic basal cell carcinomas
E. All of these answers are correct

A

►B

Basal cell nevus syndrome (Gorlin Syndrome) is caused by a mutation in the PTCH gene, located on chromosome 9q22. This gene is found to be mutated in 30-40% of sporadic basal cell carcinomas. Gorlin Syndrome is inherited in an autosomal dominant pattern. Tumors associated with this sydrome include medulloblastoma and meningioma. It is characterized by the appearance of multiple BCCs during childhood, odontogenic keratocysts of the jaw, and skeletal defects (including frontoparietal bossing and hypertelorism, among others).

178
Q

180 -What temperature must be achieved for adequate treatment of a small superficial squamous cell carcinoma with cryotherapy?

A. -10 degrees Celcius
B. -20 degrees Celcius
C. -30 degrees Celcius
D. -40 degrees Celcius
E. -50 degrees Celcius

A

►E

Cryosurgery destroys tumor if it is frozen to -40 to -70 degrees Celcius, for at least 2 cycles with a 60 second thaw. Benign lesions can be destroyed at temperatures around -25 degrees Celcius.

179
Q

181- This tumor, also called a Shagreen patch, is characteristic of which of the following genodermatoses?

A. Neurofibromatosis-1
B. Neurofibromatosis-2
C. Tuberous sclerosis
D. NAME syndrome
E. Buschke-Ollendorf

A

►C

Tuberous sclerosis a rare genodermatosis characterized by adenoma sebaceum, seizures and mental retardation. Other cutaneous features include hypopigmented macules, periungual fibromas, fibrous plaque of the face, cafe-au-lait macules and connective tissue nevus.

180
Q

182- A 30 year-old gentleman presents with multiple self healing lesions similar to the one pictured. Other family members also have the same disease. What is the most likely diagnosis?

A. Epidermodysplasia verruciformis
B. Ferguson-Smith syndrome
C. Nevoid basal cell carcinoma syndrome

D. Dyskeratosis congenita
E. Basex syndrome

A

►B

Ferguson-Smith syndrome is a rare autosomal dominant condition that is characerized by multiple keratoacanthomas found in sun-exposed areas. Lesions typically regress over weeks to months.

181
Q

183- At what location is this tumor LEAST likely to recur after surgical excision?

A. Nasolabial fold
B. Lateral canthus
C. Chin
D. Preauricular
E. Scalp

A

►E

Basal cell carcinomas are slow-growing, epithelial tumors. Highest area of recurrences after surgical excision is in the “H-zone” of the face which include nose, nasolabial folds, periocular, periauricular. The scalp is a M-zone.

182
Q

184- Which test should be used to detect monoclonal gene rearrangements in cutaneous T-cell lymphoma?

A. Northern blot

B. Southern blot
C. Western blot
D. ELISA
E. Electrophoresis

A

►B

Monoclonality and gene rearrangements can be detected with a Southern blot. Southern blots can be used to detect specific DNA fragments by gel-transfer hybridization.

183
Q

185- All of the following are true regarding lentigo maligna except:

A. High rates of recurrence
B. Occurs mostly on head and neck
C. Mostly in sun-exposed areas
D. Margins difficult to evaluate
E. Spares oral mucosa

A

►E

Lentigo maligna is a type of melanoma in situ that usually presents as a poorly circumscribed, variably colored patch. It occurs on sun-exposed areas on the head and neck. Margins may be difficult to evaluate and recurrences are relatively common. There are rare reports of spread onto the oral mucosa as well as conjunctiva.

184
Q

186- Characteristics indicative of a high risk of metastasis with SCC include:

A. Mitotic rate
B. Extremity location
C. Well differentiated
D. Deep invasion
E. Papillomavirus infection

A

►D

The rate of SCC metastasis from all skin sites ranges from 0.5% to 5.2%. Local recurrences and metastasis are related to: 1.treatment modality, 2. prior histological, 3. location, 4. size, 5. depth , 6. histological differentiation, 7. histological evidence of perineural involvement, 8. histological evidence of desmoplastic features, 9. precipitating factors other than UV light, and 10. host immunosuppression. In reference to metastatic disease, the highest rates occur from scars, the lip, and the external ear. Patients with perineural spread have a local recurrence rate of 47.2% and a metastatic rate of 34.8%. Demosplastic SCC’s are 6 times more likely to metastasize than other histological patterns.

185
Q

187- What cancer in women most commonly metastasizes to the skin?

A. Breast cancer
B. Medullary thyroid carcinoma
C. Glioblastoma multiforme
D. Colon adenocarcinoma
E. Cervical cancer

A

►A

Breast cancer is the most common cancer to metastasize to the skin in women.

186
Q

188- A young woman presents with a single small, firm, umbilicated papule on the face. Biopsy reveals a well-circumscribed lesion located in the upper dermis. Strands of basaloid cells are seen surrounded by fibrotic or desmoplastic stroma. Horn cysts and foci of sebaceous cells and calcification are also noted. How should this patient and her lesion be treated?

A. Reassurance and no further treatment
B. Close clinical follow-up
C. Topical 5-fluorouracil
D. Cryotherapy
E. Local surgical excision

A

►E

This patient has a desmoplastic trichoepithelioma. Desmoplastic trichoepithelioma is a variant of trichoepithelioma, an uncommon adnexal tumor with differentiation toward hair structures. It presents commonly in young women as a small, firm, umbilicated papule on the face. Local surgical excision is the treatment of choice.

187
Q

189- A 55 year-old female presents with an ulcerated malignant melanoma with Breslow dept of
1.5mm. Sentinel lymph node biopsy is negative. Which of the following is the correct staging classification?

A. Stage IB
B. Stage IIA
C. Stage IIB
D. Stage IIIA
E. Stage IIIB

A

►B

The current melanoma staging is developed by the American Joint Comittee on Cancer. This system is based on four characterisitics: 1. Tumor thickness 2.Presence of ulceration within the primary tumor, 3. Involvement of lymph nodes and 4.presence of distal metastases. Stage 1A characterizes a melanoma that has less than 1mm in tumor thickness and no ulceration, no nodes, and no distant mets. Stage IB characterizes a melanoma with tumor thickness less than 1mm however with ulceration and no nodes or distal metastases. Stage IB also can characterize melanomas with tumor thickness between 1.01-2.00 mm without ulceration. Stage IIA classifies tumor with thickness between 1.01-2.00mm, with ulceration, and no nodes and no distal metastases. Stage IIA also characterizes those melanomas with thickness between 2.01-4.0mm without ulceration, nodes, or distal mets. Stage III and IV melanomas have nodal and distal met involvement respectively.

188
Q

190- Which of the following is most likely to present with cutaneous metastases in men?

A. Lung cancer
B. Colon cancer
C. Prostate cancer

D. Melanoma
E. Esophageal Cancer

A

►D

The most common malignancy to present with cutaneous metastases in men is from melanoma, followed by squamous cell cancer of the head and neck. In women, breast cancer is most likely to present with cutaneous metastases. Special presentations of cutaneous metastases include alopecia neoplastica (scarring alopecia) and zosteriform metastases.

189
Q

191- A 56-year old woman with a history significant for chronic lymphedema after radical mastectomy twelve years ago presents with this growth on her arm. What is the diagnosis?

A. Angiosarcoma
B. Bacillary angiomatosis
C. Castleman’s syndrome
D. Kaposi’s sarcoma
E. Metastatic breast carcinoma

A

►A

Angiosarcoma may occur in association with chronic lymphedematous states. Stuart-Treves syndrome applies to angiosarcoma arising in an area of chronic lymphedema, like upper arm lymphedema after mastectomy.

190
Q

192- Imiquimod is an immune response modifier that stimulates innate and cell mediated immune pathways. It induces all of the following cytokines EXCEPT:

A. IL-1
B. IL-4
C. IL-5
D. IL-6
E. IL-8

A

►B

Imiquimod induces the synthesis and release of cytokines such as interleukins 1, 5, 6, 8, 10, and 12, among others.

191
Q

193- Which syndrome is characterized by multiple keratoacanthomas beginning in childhood?

A. Muir-Torre
B. Ferguson Smith
C. Rombo
D. Grybowski
E. Keratoacanthoma centrifugum marginatum

A

►B

Ferguson Smith is familial (AD), and is characterized by regressing keratoacanthomas beginning in childhood. Grybowski is a non-familial disorder in which patients develop generalized eruptive

keratoacanthomas beginning between age 40 to 60. Keratoacanthoma centrifugum marginatum is most commonly a solitary lesion occurring later in life. Muir Torre is a syndrome characterized by gastrointestinal or genitourinary neoplasms which precede development of sebaceous neoplasms and keratoacanthomas by one to two decades. Rombo is not associated with keratoacanthomas, but is associated with multiple basal cell carcinomas, atrophoderma vermiculatum, hypotrichosis, and milia.

192
Q

194- A 3-year-old girl presents with multiple small, angulated, firm nodules. There is a positive “tent sign” and biopsy reveals ghost cells and germinative cells. Of the following conditions associated with multiple pilomatricomas, which is thought to be most closely linked?

A. Turner’s
B. Myotonic dystrophy
C. Rubenstein-Taybi
D. Sarcoidosis
E. Gardner’s syndrome

A

►B

Pilomatricomas are the most common superficial pediatric tumor. The occurrence of multiple pilomatricomas has been most closely associated with the development of myotonic dystrophy(Steinert Disease). The onset of myotonic dystrophy, an autosomal dominant disorder with variable penetrance, can occur before or after the onset of lesions. Cigliano et al. reported myotonic dystrophy in 1 out of 2 patients with multiple pilomatricomas; Julian et al. reported myotonic dystophy in 1 out of 4 patients. The other syndromes associated with multiple pilomatricomas include Rubenstein-Taybi, Turner syndrome, Gardner syndrome, and sarcoidosis.

193
Q

What is the most common location for an epitheloid sarcoma?

A. head and neck
B. proximal extremities
C. hands and forearms
D. lower legs
E. groin and buttocks

A

►C

Epithelioid sarcoma is a rare soft tissue sarcoma that most commonly develops in young adults, males greater than females, with a predilection for the distal upper extremities, namely hands and forearms. Most tumors present as firm-to-hard palpable masses, either in the deep soft tissue or in the dermis. The superficial lesions can present with ulceration. Five year survival and ten year survival rate for patients with epithelioid sarcoma are approximately 50-70% and 42-55% respectively (Journal of bone and Joint Surgery (Am), 70-A: 862-870, 1988). Female patients have a more favorable outcome. Proximal lesions have been shown to have worse outcomes compared to distal lesions.

194
Q

196- What is the most common location of oral SCC?

A. Soft palate
B. Buccal mucosa
C. Gingiva
D. Dorsal tongue
E. Lateral tongue

A

►E

Squamous cell carcinomas are the most common carcinoma of the oral cavity. The most common locations for this tumor are the lateral and ventral surfaces of the tongue and the floor of the mouth.

195
Q

197- Desmoplastic trichoepitheliomas are commonly located on:

A. Legs
B. Chest
C. Back
D. Face
E. Scalp

A

►D

Desmoplastic trichoepithelioma presents as small, firm, umbilicated papule on the face of young adults.

196
Q

198- The type of keratoacanthoma that is seen in children with a autosomal dominant pattern and multiple in numbers is called:

A. Ferguson-Smith
B. Grzybowski
C. Verrucous carcioma
D. KA centrifugum
E. Buschke-Lowenstein

A

►A

Patients that have multiple keratoacanthoma in childhood have Ferguson-Smith type of KA. This is autosomal dominant and are self-healing.

197
Q

199- All of the followings can be used for treatemt of this condition except

A. Cryotherapy
B. Topical Imiquimod 5% cream
C. Topical 5-flurouracil
D. Topical retinoids
E. Surgical excision

A

►E

Attached picture is disseminated superficial actinic porokeratosis (DSAP) which is the most common type of all porokeratosis, with multiple thin papules appearing most commonly on the legs of adult women. Many treatments have been used for this condition which include: cryotherapy, topical 5-fluorouracil (5-FU), topical retinoids, CO2 laser, and dermabrasion. Although surgical excision might be used for treatment of other forms of porokeratosis, it is not advised in this case because of number of lesions and increase risk of scarring. Other forms of porokeratosis are: porokeratosis of Mibelli, punctate porokeratosis, linear, and Porokeratosis palmaris et plantaris disseminata.

198
Q

200- What is the most common location for a fibroepithelioma of pinkus variant of BCC is

A. head and neck
B. extremities
C. hands and feet
D. upper trunk
E. lumbosacral region

A

►E

Fibroepithelioma of pinkus variant of BCC is often a fleshy or pink colored nodule. The most common location of a fibroepithelioma of pinkus is the lumbosacral region. Superficial variants are more common on the trunk and extremities. BCC makes up the most common non melanotic skin cancer at 75%.

199
Q

201- Merkel cell carcinoma has been found to be associated with which of the following viruses?

A. Herpes virus
B. Polyomavirus
C. Paramyxovirus
D. Flavivirus
E. Enterovirus

A

►B

Merkel cell carcinoma has been found to be associated with the merkel cell polyomavirus (MCPyV). Herpes virus causes diseases such as HSV, VZV, EBV, CMV, roseola, and Kaposi’s sarcoma. Paramyxovirus causes measles. Flavivirus causes diseases such as west nile virus, dengue, and yellow fever. Enteroviruses are not associated with merkel cell carcinoma.

200
Q

202 - The Grzybowski type of keratoacanthoma:

A. Is characterized by rapid growth of a single lesion reaching a diameter of 9 cm or more
B. Typically invades underlying cartilage
C. Demonstrates simultaneous central healing
D. Presents in childhood on sun-exposed surfaces
E. Presents with hundreds of disseminated lesions

A

►E

The Grzybowski type of keratoacanthoma is typically diagnosed in adulthood, with the sudden appearance of hundreds of lesions in a disseminated fashion. The lesions are generally 2 -3mm in diameter and can be found anywhere on the body including the palms, soles, larynx, and oral mucosa.

201
Q

202- Which of the following are the most reliable prognostic factors in malignant melanoma?

A. Breslow‘s depth and ulceration
B. Breslow‘s depth and Clark level
C. Clark level and ulceration
D. All of these answers are correct
E. None of these answers are correct

A

►A

The most reliable prognostic factors in MM are Breslow‘s depth and ulceration of the primary tumor. Breslow depth is the thickness of the melanoma measured from the granular layer to the deepest point of tumor invasion.

202
Q

203 -Patients that are organ transplant patients have a higher risk of developing skin cancer such as squamous cell carcinoma. The risk can be as much as:

A. 65x
B. 50x
C. 40x
D. 25x
E. 10x

A

►A

The risk of SCC in an organ transplant population is elevated by 65x. These patients should have a year skin examination.

203
Q

204- The Pinkus BCC is frequently located on the:

A. Digits
B. Eyelid
C. Lips
D. Ear
E. Lumbosacral area

A

►E

Pinkus BCC (also known as fibroepithelioma) appears as a pedunculated, dome-shaped papule frequently located on the back (lumbosacral area).

204
Q

205- What is the average time between a BCC primary tumor and its metastasis?

A. 6 months
B. 1 year
C. 2-4 years
D. 9 years
E. greater than 15 years

A

►D

The size of a BCC is related to its metastasis risk. Invasion of bone, cartilage, and muscle is not common, however, spread does occur along perichondrium, periosteum, fascia, or tarsal plate.

205
Q

206- All of the following statements regarding the patient pictured (BCC syndrome) are true EXCEPT:

A. Spina bifida may be an associated finding
B. The patient likely has a mutation in the PTCH gene
C. Pheochromocytoma is a tumor associated with this disease
D. This patient likely had a similarly affected parent
E. These lesions appeared in childhood

A

►C

The patient has basal cell nevus syndrome, or Gorlin syndrome, which is characterized by the appearance of multiple basal cell carcinomas in childhood, odontogenic keratocysts of the jaw, and skeletal defects (i.e macrocephaly, hypertelorism, frontoparietal bossing, spina bifida, or rib abnormalities). Tumors associated with this disease include medulloblastoma and meningioma (not pheochromocytoma). It is caused by a mutation in the PTCH gene located on 9q22 and is inherited in an autosomal dominant manner.

206
Q

207- Which of the following neoplasms has demonstrated an association with HTLV-1 infection?

A. Mycosis fungoides
B. Adult T-cell lymphoma
C. Follicular lymphoma
D. Multiple myeloma
E. Hodgkin‘s disease

A

►B

Mycosis fungoides is a rare form of cutaneous T-cell lymphoma, the etiology of which is not completely known. Adult T Cell Lymphoma is also a type of T cell neoplasm that has been linked to

HTLV-1. Adult T-cell lymphoma may have an acute and chronic, smoldering form. The chronic, smoldering form can be difficult to distinguish from mycosis fungoides.

207
Q

208- Mohs micrographic surgery the treatment of choice for all of the following, except:

A. 1 cm SCC located on the chest
B. 2 cms BCC on lower extremities
C. 1 cm BCC on the eyelid
D. Morpheaform BCC on the cheek
E. Recurrent BCC on the chest

A

►A

SCCs that measure 1cm on the chest are not an indication for Mohs micrographic surgery. Lesions must measure 2cms on the chest and extremities, or 1cm for those located on the face to be considered for Mohs surgery. Due to the aggressive growth patter and subclinical spread of morpheaform and recurrent BCCs are always and indication for Mohs surgery.

208
Q

209- This type of verrucous carcinoma is locally aggressive but rarely metastasizes and is on the plantar area is:

A. Epithelioma cuniculatum
B. Buschke-Lowenstein
C. Oral florid papillomatosis
D. Subungual
E. Giant SCC

A

►A

Epithelioma cuniculatum is a plantar SCC that is locally aggressive but rarely metastasizes, Oral florid papillomatosis is the a type that is found in the mouth. Buschke-Lowenstein is the type that you find in the genital area.

209
Q

210- Which is the most common neoplasm in patients who have had long-term PUVA therapy?

A. Basal cell carcinoma
B. Atypical fibroxanthoma
C. Squamous cell carcinoma
D. Cutaneous T cell lymphoma
E. Melanoma

A

►C

The most common neoplasm to arise in patients who have been treated with long -term PUVA therapy is squamous cell carcinomas. In a study by Stern, et.al. a 12.8 fold risk was found for the development of squamous cell cancers in patients who received high doses than those that received low doses.

210
Q

211- Which of the following markers do not stain melanocytic lesions:

A. CK7
B. Vimentin
C. S-100
D. HMB-45
E. All of these answers are correct

A

►A

CK7 will stain keratinocytes. Vimentin, S-100 and HMB-45 stains melanocytic lesions.

211
Q

Which of the following are features of microcystic adnexal carcinoma that help distinguish it from desmoplastic trichoepitheliomas?

A. Deep subcutaneous infiltration
B. Perineural invasion
C. CEA positive staining
D. Commonly located on the face
E. Deep subcutaneous infiltration,perineural invasion, and CEA positive staining

A

►E

Desmoplastic trichoepitheliomas is one of the differential diagnosis for MAC. MAC show deep subcutaneous and perineural invasion, as well as CEA positive staining, all features that may help differentiate it from desmoplastic trichoepitheliomas. Both, MAC and desmoplastic trichoepitheliomas are commonly located on the face.

212
Q

-Which of the following is a common location of melanoma in women?

A. Chest
B. Lower legs
C. Genitals
D. Digits
E. Scalp

A

►B

The most common locations of MM in women are the back, lower legs and upper extremities.

213
Q

214 -The most common location of superficial spreading melanoma in men is:

A. Lower legs
B. Back
C. Upper extremities
D. Head and neck
E. Digits

A

►B

Superficial spreading melanoma may be located anywhere, but the back is the most common site in male patients.

214
Q
  1. Bazex syndrome can be differentiated clinically from Rombo syndrome by presence of:

A. Multiple basal cell carcinomas
B. Trichiepitheliomas
C. Milia
D. Follicular atrophoderma
E. Hypohidrosis

A

►E

Localized hypohidrosis is a feature found in Bazex syndrome but not Rombo syndrome. Another differentiating feature is that Rombo syndrome classically has vermiculate atrophoderma, while Bazex has follicular atrophoderma.